Anda di halaman 1dari 63

Assessment Exam for

USMLE Step 3: *

Internal Medicine
Answers and Explanations

*USMLE is a joint program of the Federation of State Medical Boards of the United States, Inc. and the National Board of
Medical Examiners.
©2004 Kaplan, Inc.

All rights reserved. No part of this book may be reproduced in any form, by photostat,
microfilm, xerography or any other means, or incorporated into any information
retrieval system, electronic or mechanical, without the written permission of Kaplan, Inc.

Not for resale.


Internal Medicine Assessment Exam

1. A 65-year-old man with a history of diabetes mellitus 2. A 52-year-old alcoholic man comes to the emergency
type 2 comes to the clinic for a routine followup appoint- department (ED) with two episodes of hematemesis
ment. Over the past year he has noticed that he has had following a drinking binge. While in the ED, the patient
numerous episodes of weakness, lightheadedness, and has two more episodes of grossly bloody emesis. At this
sweatiness. Twice he has lost consciousness. At these times time the patient’s vital signs are remarkable for a blood
he has been found to have a low blood glucose level, and pressure of 92/60 mm Hg and a pulse of 120/min. The
his symptoms quickly improve with the administration of patient is disoriented and combative and unable to give
oral glucose gel or a sugary snack or piece of candy. The a coherent history, though he is well known to many of
man denies being ill, changing his diet or eating habits, or the emergency department staff. Physical examination
increasing his insulin dose. Indeed, he has been on the is remarkable for multiple spider angiomata, palmar
same dose of twice-daily NPH insulin for the last 3 years.
erythema, and hepatomegaly by percussion. Laboratory
Further, before this last year or two, the patient had found
studies show:
it extremely difficult to control blood glucose levels.
Long-standing diabetes has resulted in diabetic retinopa-
thy, a silent myocardial infarction 1 year ago, erectile dys- Hemoglobin 8 g/dL
function, and peripheral neuropathy. Today, vital signs Hematocrit 29%
and physical examination are unremarkable. Recent labo- Platelets 123
ratory studies show:
PT/INR 28 s/1.8
Blood, Plasma, Serum aPTT 103 s
Bicarbonate 24 mEq/L WBC 4500/mm3
Chloride 105 mEq/L
Two 16-gauge peripheral intravenous lines are placed,
Creatinine 2.7 mg/dL and a transfusion of fresh frozen plasma and blood is
Potassium 5.2 mEq/L begun. An infusion of a somatostatin analogue is begun
Sodium 131 mEq/L simultaneously, and the patient is transferred to the
Urea nitrogen 28 mg/dL intensive care unit. An emergent upper endoscopy
reveals bleeding esophageal varices that are treated suc-
Hematologic cessfully with rubber band ligation. Over the next couple
Hemoglobin A1c 4.9% of days the patient recovers and is prepared for discharge.
In addition to recommending abstinence from alcohol,
Of note, the patient’s hemoglobin A1c 1 year ago was 8% an appropriate medical treatment likely to reduce this
and 2 years ago was 9.5%. A urinalysis is remarkable for patient’s risk for recurrent bleeding is
2+ proteinuria. A chest radiograph is unrevealing,
whereas an electrocardiogram reveals Q waves in leads II, (A) famotidine
III, and aVF. Given this patient’s presentation, the most (B) misoprostol
likely cause of his repeated episodes of hypoglycemia is (C) octreotide
(A) development of insulinoma (D) omeprazole
(B) heart failure (E) propranolol
(C) noncompliance
(D) renal failure
(E) resolution of underlying diabetes

1
USMLE Step 3 Assessment Exam

3. A 70-year-old man comes to the office complaining of 4. A 35-year-old man is rushed to the emergency depart-
knee and ankle pain. For the last couple of years he has ment after crashing his car into a tree late one Saturday
suffered from pain and swelling in his right knee and night. He sustains a femur fracture requiring an open
ankle and has recently developed similar pain in his left reduction and internal fixation. The operation goes well
elbow. This winter the pain has been particularly severe, and the patient is sent to the surgical step-down unit for
prompting the man to seek help. The pain does bother recovery. Over the next 2 postoperative days, he becomes
the man in the morning, when he feels stiff, but is usu- increasingly agitated and confused. He often does not
ally worse at the end of the day. He denies any associat- know where he is and suffers hallucinations and delu-
ed fevers, rash, or back pain. Past medical history is sig- sions, seeing insects crawling up the walls and mistaking
nificant for coronary artery disease, high blood pres- shadows for devils. Laboratory studies are remarkable
sure, and a gastric ulcer. Current medications include a for a mild transaminitis, including an elevated GGT, a
daily baby aspirin, a beta-blocker, and an H2 receptor mild macrocytic anemia with a hematocrit of 36%, and
blocker. Vital signs are normal, but examination of the a mild hypokalemia and hypomagnesemia. Near the end
patient’s hands reveals mild swelling and nodularity of of postoperative Day 2, the patient suffers a short gener-
multiple distal interphalangeal joints of the right hand. alized seizure that resolves before the house officer on
Crepitation is present in the affected elbow and knee. call arrives at the scene. Given this patient’s presenta-
Although the joints are mildly swollen, they are not ten- tion, the most appropriate next step in this patient’s
der or erythematous. Reviewing the patient’s past labo- management is to
ratory studies reveals that he recently has had a normal
(A) begin treatment with broad-spectrum antibiotics
erythrocyte sedimentation rate and, aside from a slight
normocytic anemia, has had unremarkable routine lab- (B) begin treatment with lorazepam
oratory studies. Given this patient’s presentation, the (C) provide daily beta-blocker and clonidine
most appropriate treatment for control of his pain is to (D) return to operating room for repair/replacement
(A) add acetaminophen of hardware
(B) begin capsaicin cream (E) start prophylactic phenytoin
(C) increase aspirin dose
(D) start an analgesic dose of ibuprofen (1200 mg/d)
(E) start an anti-inflammatory dose of ibuprofen
(2400 mg/d)

2
Internal Medicine

5. A 66-year-old man comes to the emergency department 6. A 24-year-old Mormon comes to the clinic complaining
complaining of chest pressure. He is visibly sweaty and, of fatigue. He recently returned from a mission abroad,
between breaths, explains that for the last 40 minutes he where he had spread the gospel through central Africa.
has suffered band-like pressure across his left chest, Since returning from abroad, he has suffered progressive
with some associated pain in his neck and shoulder. The fatigue to the point that he is no longer able to ride his
patient has never had pain like this before, but he has a bicycle for more than a block without needing to rest.
history of high blood pressure and diabetes and his Additionally, friends have remarked that, despite
father died of a heart attack at age 58 years. The patient months in the tropical sun, he appears quite pale. Over
receives oxygen and aspirin and is placed on a moni- the last 2 weeks the man also has noticed numerous
tored bed. Vital signs are temperature of 37.4 C (99.3 bruises on both of his legs. The young man denies any
F), blood pressure of 144/88 mm Hg, pulse of 110/min, recent fevers, chills, headaches, weight loss, night sweats,
and respirations of 28/min. Physical examination or other systemic symptoms. The patient has not had
reveals some mild chest wall tenderness, but is other- any recent respiratory infections, though he did have a
wise unremarkable. The pain improves with some severe case of conjunctivitis in Africa, which was treated
nitroglycerin, and stat laboratory studies are sent. An with chloramphenicol eye drops. The conjunctivitis has
electrocardiogram is performed, and a concerned nurse since resolved, and he has not taken any medications,
points out several abnormalities. An electrocardio- topical or otherwise, in months. A careful review of
graphic finding that would justify thrombolytic therapy social and sexual habits reveals no drug use or high-risk
in this patient is sexual behavior. Family history is remarkable for an
uncle with prostate cancer and a grandfather who died
(A) diffuse T-wave flattening or inversions in all leads
of colon cancer. Physical examination, aside from
(B) new-onset left bundle branch block seen in pre- marked pallor and some lower extremity ecchymosis, is
cordial leads within normal limits. A complete blood count shows:
(C) new Q waves in lead 2, lead 3, and lead aVF
(D) new R-R′ pattern in precordial leads Hematologic
(E) ST segment depression of >1 mm in the inferior/ Hemoglobin 6.9 g/dL
lateral leads Leukocyte count 2000/mm3
Platelets 38,000/mm3
Reticulocytes 0.2%

A chest radiograph and urinalysis are normal. The


peripheral smear, aside from a marked pancytopenia,
shows no abnormal cells or immature forms. Red cells
appear normocytic, there are no platelet clumps, and
few reticulocytes are seen. A bone marrow biopsy is
performed that shows marked hypocellularity with dif-
fuse fatty infiltration. Given this patient’s presentation,
the most appropriate management is
(A) evaluation for allogenic bone marrow transplan-
tation
(B) initiation of antithymocyte globulin and/or
cyclosporine
(C) treatment with granulocyte colony-stimulating
(GCSF) factor
(D) treatment with weekly injections of erythropoietin
(E) watchful waiting and blood product transfusions
as necessary

3
USMLE Step 3 Assessment Exam

7. Family members bring a 38-year-old woman to the 8. A 22-year-old homeless man arrives in the emergency
emergency department. The family members state that department complaining of fever, fatigue, a 2-week his-
the woman suddenly became confused, nonresponsive, tory of a cough, and occasional hemoptysis. A quick
and developed a right-sided facial droop. Furthermore, physical examination is remarkable for a temperature
approximately a month ago she was diagnosed with of 38.0 C (100.4 F), a faint diastolic murmur that
leukemia after a history of unexplained fevers and increases with inspiration, and obvious needle marks
weight loss. At that time she was started on all-trans- on the patient’s arms and legs. Without much resis-
retinoic acid. A quick evaluation in the emergency tance, the patient admits to prolonged drug and alcohol
department reveals a confused woman oriented only to use, including inhaled crack cocaine and intravenous
self, with left-sided hemiparesis and a left-sided heroine. Subsequent workup includes a chest radio-
Babinski sign. A stat head CT scan confirms the fears of graph, a transthoracic echocardiogram, three sets of
the emergency department attending physician; the blood cultures before antibiotics, and an electrocardio-
woman has suffered a right intraparenchymal hemor- gram. Results are as follows:
rhage. Laboratory studies show:
Chest radiograph Multiple cavitating
Hematologic nodular lesions
Hemoglobin 8.0 g/dL Echocardiogram Tricuspid vegetation
Leukocytes 2200/mm3 and regurgitation,
Platelets 52,000/mm3 normal ventricular
function
Prothrombin time 22 seconds
Partial thromboplastin time 55 seconds Blood cultures Highly resistant
methicillin-resistant
Additional laboratory studies reveal an elevated D- Staphylococcus aureus
dimer and reduced fibrinogen level. A peripheral smear (MRSA)
shows expected atypical cells with the presence of Auer Electrocardiogram Prolonged PR-interval,
rods and occasional schistocytes and helmet cells. Given otherwise within normal
this patient’s presentation, the form of leukemia she is limits
suffering from is
(A) M0undifferentiated Based on this patient’s presentation, the most com-
pelling reason to request a surgical consultation is
(B) M3promyelocytic
(C) M5monocytic (A) cultures indicating highly resistant organism
(D) M6erythroleukemic (B) no surgical treatment indicated at this time
(E) M7megakaryoblastic (C) presence of electrocardiogram abnormalities
(D) radiographic evidence of embolic events
(E) right-sided lesion associated with regurgitation

4
Internal Medicine

9. A 58-year-old man comes to the clinic complaining of 10. A 34-year-old man comes to the office for follow-up for
weakness and leg pain. Approximately 10 days ago he an asthma attack. He presented to a local emergency
was started on ampicillin/sulbactam for treatment of department wheezing and responded to beta-agonists
cellulitis, believed to be caused in part by a small ulcer- and steroids. Pulmonary function tests at that time indi-
ation between his toes. Since then his cellulitis has cated an obstructive ventilatory defect and a low peak
resolved and he has completed the antibiotics this flow consistent with acute bronchospasm. He has not
morning as prescribed. For the past 2 days he has felt had asthma before, but over the past few years he has
slightly confused, has suffered generalized malaise and developed severe episodes of nasal congestion and rhin-
weakness, and has had crampy pain in his legs. He orrhea, which he had assumed were caused by “hay
denies any fevers, rigors, cough, dysuria, or constitu- fever.” The only thing this man can think of that may
tional symptoms. Vital signs are: temperature 37.0 C have triggered these past attacks and his recent episode is
(98.6 F), blood pressure 178/98 mm Hg (at previous the occasional use of ibuprofen or aspirin. Head and
visit, 138/88 mm Hg), pulse 105/min, and respirations neck examination is remarkable for some mild rhinor-
20/min. Cardiac examination reveals a jugular venous rhea, boggy turbinates, and the presence of two small
pressure of 10 cm/H2O, a regular tachycardia without nasal polyps. Chest auscultation is unremarkable; there
murmurs, and the presence of a rub. Auscultation of are no wheezes or crackles. The rest of the examination is
the chest reveals faint bibasilar crackles, and there is 1+ normal. Given this patient’s presentation, an appropriate
pitting edema bilaterally. A maculopapular rash is pre- treatment for his symptoms, in addition to a beta-agonist
sent across the patient’s back. The rest of the examina- rescue inhaler and the avoidance of aspirin, is
tion is unremarkable. Laboratory studies show:
(A) aminophylline or theophylline
Blood, Plasma, Serum (B) epi-pen (SQ emergency epinephrine)
Creatinine, serum 6.9 mg/dL (C) nedocromil meter dose inhaler
Electrolytes (D) salmeterol meter dose inhaler
Sodium 135 mEq/L (E) zafirlukast or montelukast
Potassium 6.2 mEq/L
Chloride 108 mEq/L
Bicarbonate 14 mEq/L
Urea nitrogen, serum 88 mg/dL

Past laboratory studies indicate that the patient has


chronic renal insufficiency with a baseline creatinine of
2.8 mg/dL. A chest radiograph shows bilateral small
effusions and a normal appearing cardiac silhouette.
Additional workup is ordered that includes a urinalysis,
complete blood count, and a Wright stain for
eosinophils. The complete blood count is remarkable for
a high percentage (14%) of eosinophils. The urinalysis
reveals 2+ proteinuria, and the Wright stain is positive
for eosinophils. Based on this patient’s presentation, the
most appropriate treatment for his condition is to
(A) administer a fluid bolus to improve perfusion
(B) restart broad-spectrum antibiotics
(C) send him for immediate hemodialysis
(D) send him for percutaneous renal biopsy
(E) start a furosemide drip

5
USMLE Step 3 Assessment Exam

11. A concerned wife and son accompany a 71-year-old 12. A 44-year-old man comes to the walk-in clinic com-
man to the clinic. They state that the man has “not been plaining of fever, fatigue, and anorexia. For the past few
acting himself ” over the past 6 months and seems to months, the patient has been homeless and has basical-
have begun his decline a year ago. They are concerned ly been on one long, occasionally interrupted alcoholic
that the patient often seems to have difficulty finding binge. Over the last week, he has noticed that his skin
words, is frequently confused, and now gets lost walk- has a yellow tint. Vital signs are temperature of 38.3 C
ing home from the grocery store. The man in response (101.0 F), blood pressure of 107/60 mm Hg, pulse of
to these claims states that he is fine, though he does 105/min, and respirations of 14/min. Physical examina-
admit that his memory is not what it used to be. He tion reveals a disheveled, cachectic-appearing man who
denies any symptoms of depression and states he would smells intoxicated. He has mild scleral icterus and is
be fine if his family would quit imagining things. Before noticeably jaundiced. Lungs are clear to auscultation
this past year, the patient has been healthy most of his and his heart has a regular rate and rhythm. His
life and has had no major illnesses, head trauma or falls, abdomen is nondistended but is mildly tender in the
dietary deficiencies, or history of high blood pressure, right upper quadrant. The liver tip is 7 cm below the
diabetes, or strokes. Vital signs and general physical costal margin by percussion. There is no splenomegaly.
examination are within normal limits. A mini-mental Shifting dullness is not present. There is no palmar ery-
status examination, however, reveals dramatic deficits thema, spider angiomata, asterixis, or other stigmata of
in concentration and recent memory. A review of the chronic liver disease. Laboratory studies show:
patient’s medical records reveals a recent normal TSH
Blood, Plasma, Serum Hematologic Serology/Toxicology
level and normal routine laboratory studies. Based on
ALT 89 U/L PT 68 sec Acetaminophen Negative
this patient’s presentation, the most appropriate next
Alk phos 172 U/L INR 4.6 HBsAg Negative
test to diagnose his current condition is
AST 298 U/L PTT 70 sec HBcAg Negative
(A) brain biopsy of temporal lobe Bili (total) 10.0 mg/dL Anti-HBC Negative
(B) central nervous system acetylcholine level
A urinalysis and a chest radiograph are within normal
(C) magnetic resonance imaging of the brain limits. An ultrasound of the abdomen reveals an
(D) no additional testing necessary enlarged liver but no evidence of ascites. In addition to
(E) referral to neurology for neurodiagnostic studies abstinence from alcohol, the most appropriate interven-
tion that will likely improve this patient’s prognosis is
(A) albumin
(B) ceftriaxone
(C) octreotide
(D) prednisolone
(E) propranolol

6
Internal Medicine

13. A 34-year-old man arrives at the office seeking help for 14. A 62-year-old woman comes to the clinic for a routine
a 4-month history of joint pain and a strange rash that physical examination. She has no significant complaints
has developed over a similar period of time. The patient and reports that she feels she is in excellent health. Her
reports pain in his right ankle and left knee that is par- past medical history is significant for diabetes mellitus
tially relieved with ibuprofen. Additionally, a strange type 2 diagnosed 3 years ago. Since then she has
rash that he originally treated with an over-the-counter achieved good glycemic control with diet, exercise, and
antifungal cream is present on the soles of his feet. He a low dose of metformin. She has had no complications
denies any previous history of joint pain or trauma and related to her diabetes; she has no history of cere-
has had no fevers, chills, weight loss, or other constitu- brovascular or coronary events, has an undetectable
tional symptoms. Vital signs are normal. Examination microalbuminuria, and has had unremarkable ophthal-
reveals a mild redness of both eyes and a mild photo- mologic and podiatric examinations. Vital signs and
phobia. Cardiac and chest examinations are normal. physical examination are within normal limits.
Palpation of the joints, particularly of the lower extrem- Laboratory studies show:
ities, reveals some mild pain at the enthesis (site of
insertion of the ligaments, tendons, joint capsule, or Blood, Plasma, Serum
fascia to bone), but no marked erythema or crepitation. Cholesterol
The soles of the patient’s feet reveal thick scaly lesions
Low-density lipoprotein 131 mg/dL
and macules and papules. The toenails are markedly
thickened, whereas the fingernails seem normal. Given High-density lipoprotein 43 mg/dL
this patient’s presentation, the most important further Triglycerides 120 mg/dL
aspect of his history to obtain is Fasting glucose 129 mg/dL
(A) family history of malignancy Hematologic
(B) family history of skin disease Hemoglobin A1c 6.8%
(C) occupational history and exposures
(D) previous dermatologic complaints Given this patient’s presentation, the most appropriate
course of action regarding her lipid profile is to
(E) recent sexually transmitted diseases (STDs) or
gastrointestinal illnesses (A) begin therapy with simvastatin, recheck lipid panel
in 3 to 6 months
(B) encourage exercise and dietary changes, recheck in
3 to 6 months
(C) increase metformin, recheck hemoglobin A1c and
lipids in 3 to 6 months
(D) no further action at this time, recheck lipid panel
in 1 year
(E) start therapy with gemfibrozil, recheck lipid panel
in 3 to 6 months

7
USMLE Step 3 Assessment Exam

15. A concerned nurse calls you at 2 AM to come see a 16. A 74-year-old woman is sent to the emergency depart-
patient complaining of shortness of breath. The patient ment after being found in a confused state at her long-
is a 77-year-old man recovering from pneumonia on term care facility. For the past 2 weeks she has been act-
the medicine floor. He reports the sudden onset of ing “not quite herself.” According to nursing staff at the
shortness of breath and that he has some mild pain and facility, she has had a cough that seems worse during
a funny feeling in his chest. He also reports feeling meals, and occasional low-grade fevers, though no rig-
incredibly weak and apparently became dizzy and fell ors. Before this episode the patient had been fairly func-
when trying to get out of bed to use the bathroom. Vital tional but required physical therapy and nursing for a
signs at this time are temperature of 38.0 C (100.4 F), recent stroke and resulting left-sided hemiparesis. Vital
blood pressure of 82/45 mm Hg, pulse of 170/min, and signs on time of arrival are temperature of 38.0 C
respirations of 22/min. Examination reveals an irregu- (100.4 F), blood pressure of 108/68 mm Hg, pulse of
lar heartbeat without rubs, gallops, or jugular venous 108/min, and respirations of 24/min. An oxygen satura-
distention. Lung fields are clear and there is no periph- tion obtained with the patient’s vital signs is 85%. The
eral edema or cyanosis. The rest of the examination is woman is confused and oriented only to self. She has a
within normal limits. An electrocardiogram reveals an cough and produces some foul-smelling green sputum.
irregularly irregular heart beat with p-waves with dif- Physical examination is remarkable for inspiratory
fering morphologies. The most appropriate treatment crackles heard over the right mid lung zone and right
for this patient’s condition is lung base. Tactile fremitus is increased in this area and
bronchial breath sounds are also present. Laboratory
(A) amiodarone bolus and continuous infusion
studies show:
(B) either intravenous diltiazem or metoprolol
(C) emergent cardioversion at the bedside Hematologic
(D) placement of a transvenous pacemaker, followed Leukocyte count 20,000/mm3
by a permanent pacemaker Neutrophils, segmented 70%
(E) urgent transesophageal echocardiogram, anticoag- Neutrophils, band 18%
ulation, and cardioversion
Hematocrit 35%
Platelet count 368,000/mm3

A urinalysis and routine set of chemistries are un-


remarkable. A chest radiograph reveals a patchy,
ill-defined opacity in the superior segment of the right
lower lobe, with a small area of central cavitation. Given
this patient’s presentation, the most appropriate treat-
ment is to
(A) begin trimethoprim/sulfamethoxazole and pred-
nisone
(B) obtain high-resolution CT scan of the lung
(C) order bronchoscopy with biopsy
(D) prescribe empiric ceftriaxone and azithromycin
(E) start empiric clindamycin

8
Internal Medicine

17. An 18-year-old woman comes to the office complaining 18. An 82-year-old nursing home patient comes to the clin-
of recent bruising, severe nosebleeds, and heavy men- ic for a routine healthcare visit. Although he has been
strual periods over the last few weeks. Before this visit, relatively healthy over the past year, he has had some
she has been healthy and, aside from a “bad case of the mild weight loss over the course of the year. The physi-
flu” a month ago, cannot recall any recent illnesses or cian who makes weekly rounds at the nursing home
symptoms. She specifically denies any fatigue, fevers, noticed that the patient appeared somewhat gaunt and,
chills, nausea, abdominal pain, or weight changes. The on physical examination, noticed some “lumps and
woman does not take any medications and knows of no bumps” in the patient’s neck. Labs were drawn and the
illnesses that run in her family. Vital signs are tempera- patient was advised to follow up with his primary care
ture of 37.0 C (98.6 F), blood pressure of 118/78 mm physician. The patient denies any fevers, chills, night
Hg, pulse of 96/min, and respirations of 14/min. sweats, abdominal pain, recurrent infections, change in
Physical examination is remarkable for a small amount bowel habits, or history of bleeding. Physical examina-
of dried blood on her nasal turbinates and multiple tion is remarkable for bilateral cervical and supraclavic-
petechiae on her buccal mucosa and on both lower ular adenopathy. The nodes are firm, mobile, and non-
extremities. There is no hepatosplenomegaly and the tender, with the largest node being approximately 2.5
examination is otherwise within normal limits. cm. Cardiopulmonary examination is normal, but pal-
Preliminary laboratory studies show: pation of the abdomen reveals some mild splenomegaly.
Laboratory study includes normal chemistries, liver
Hematologic function tests remarkable only for a mild hypogamma-
Hematocrit 37% globulinemia, and normal thyroid function tests. A
complete blood count, however, shows:
Leukocyte count 10,000/mm3
Neutrophils 60% Hematologic
Lymphocytes 30% Hemoglobin 12.5 g/dL
Platelets 18,000/mm3 Leukocytes 48,200/mm3
Prothrombin time 10 seconds Neutrophils 10%
Partial thromboplastin time 25 seconds Lymphocytes 84%
Monocytes 5%
A manual review of the blood smear by the laboratory Platelets 152,000/mm3
technician reveals some large platelets and rare platelet
clumps, but no abnormal cells or schistocytes. Given A review of the blood smear reveals a predominance of
this patient’s likely illness, the most appropriate first mature lymphocytes. The other cell lines seem to have a
line treatment is normal morphology. Based on this abnormal smear, the
(A) DDAVP or factor VIII replacement patient undergoes a bone marrow biopsy and has blood
sent for flow cytometry (cell marker studies). The bone
(B) immediate platelet transfusion
marrow smear shows a diffuse infiltration with mature-
(C) intravenous gamma-globulin (IVIg) appearing lymphocytes and an increased number of
(D) 1 month of prednisone with taper megakaryocytes. Myeloid and erythroid cell lines seem
(E) transfusion of fresh frozen plasma normal. Flow cytometry reveals a predominance of
CD20 and CD5 positive cells. Based on this presentation,
the most appropriate treatment plan for this patient is to
(A) begin treatment with high-dose chlorambucil
(B) begin treatment with low-dose chlorambucil
(C) follow up as needed or in 1 year, no treatment
(D) repeat CBC and examination in 3 months, no
treatment
(E) start pulse prednisone with prolonged taper

9
USMLE Step 3 Assessment Exam

19. A 55-year-old man comes to the office complaining of a 20. A 53-year-old, obese man undergoes spinal surgery for
swollen left leg. The condition started approximately 2 multilevel spinal stenosis. The operation goes well but is
days ago and seems to be getting worse. He denies any technically challenging, requiring a prolonged operat-
recent immobilization, shortness of breath, fever, chills, ing room course of more than 12 hours. During this
or pleuritic pain. The man does, however, report a past time, the patient is kept in the prone position to facili-
history of coronary artery disease, high blood pressure, tate laminectomy. Postoperatively, the patient is moni-
and atrial fibrillation. Current medications include war- tored in the ICU, where he is noted to have dark, rusty
farin and metoprolol. Inspection of the leg reveals an old urine and unexpectedly low urine output. Vital signs
scar that the patient states is from a previous vein graft are: temperature 36.8 C (98.2 F), blood pressure 120/77
for a coronary artery bypass 2 years ago and a distinct mm Hg, pulse 100/min, and respirations 20/min. The
area of erythema. The patient’s foot to mid-calf is warm, patient is intubated, but when sedation is weaned, he is
erythematous, and tender to touch. The patient’s feet alert and oriented, follows commands, and does not
show evidence of tinea, but the lower extremities are oth- seem to be in any pain. Aside from diffuse muscu-
erwise unremarkable. Given this patient’s presentation, loskeletal tenderness over pressure points from the
the most appropriate next step in his management is operating room, the examination is unremarkable. A
quick urine dipstick reveals 3+ blood. Concerned about
(A) complete blood count, set of chemistries, and
rhabdomyolysis, the intern taking care of the patient
blood cultures
sends off a full panel of chemistries, a microscopic uri-
(B) duplex ultrasonography of the leg nalysis, an arterial blood gas, and a creatine phosphok-
(C) empiric antibiotics for gram-positive and gram- inase level. An expected result of these tests would be a
negative organisms
(A) high arterial blood pH
(D) skin biopsy and culture of the edge of the erythe-
(B) high urine red blood cell count
matous lesion
(C) low serum calcium
(E) venography of lower extremity
(D) low serum phosphate
(E) low serum potassium

10
Internal Medicine

21. A 22-year-old man comes to the office seeking treatment 22. A 25-year-old man is brought to the emergency depart-
for a cough and wheeze. His symptoms occur only inter- ment in respiratory distress. He is brought in on a
mittently and usually are not too severe. However, he stretcher, as he has been unable to get out of bed for the
recently went skiing, and had a severe episode requiring last 5 days. A quick look at the man reveals that he is
nebulized treatments at an urgent care center. The man having difficulty breathing. While speaking, he often
sheepishly admits to smoking half a pack a day and real- gets winded and has to stop and catch his breath. He
izes it may worsen his symptoms. His recent episode has relates a recent history of weakness that started in both
convinced him to quit, but he is worried that his symp- of his ankles and, over the course of the week, extended
toms will recur. The young man is otherwise quite to include his legs, then hips, and finally his torso.
healthy. Physical examination reveals a well-appearing Today he has had extreme difficulty breathing and
man in no apparent distress. Vital signs are: temperature states that the process is “exhausting.” Aside from a
36.7 C (98.0 F), blood pressure 124/88 mm Hg, pulse recent episode of gastroenteritis characterized by gross-
68/min, and respirations 16/min. Auscultation of the ly bloody diarrhea, the man has been healthy. He denies
chest reveals no crackles, wheeze, or bronchial breath any systemic illnesses, intravenous drug use, or HIV
sounds, and tactile fremitus is normal. The rest of the risk factors, though he does smoke marijuana and occa-
examination is unremarkable. A chest radiograph sionally binge drinks. Physical examination is remark-
reveals clear lung fields and crisp, well rounded able for flaccid paralysis of the lower extremities with
diaphragms. The patient also receives evaluation with no deep tendon reflexes. Sensation is mildly reduced in
spirometry and carbon monoxide diffusion testing. the lower extremities and torso, but cranial nerve exam-
Given this patient’s presentation, the most likely finding ination is within normal limits. In addition to respira-
on pulmonary function tests (PFT) is tory and other supportive care, an appropriate treat-
ment for this patient’s condition is
(A) an abnormal elevation of flow rates using a peak
expiratory flow meter (A) antibiotics
(B) an FEV1/FVC ratio of less than 60 to 70% of pre- (B) antitoxin
dicted values (C) interferon
(C) an increase in the diffusing capacity for carbon (D) plasmapheresis
monoxide (DLCO)
(E) steroids
(D) normal, with obstructive pattern following metha-
choline challenge
(E) a reduction in the total lung capacity, with pre-
served FEV1/FVC ratio

FEV = forced expiratory volume


FEV1 = forced expiratory volume in 1 second

11
USMLE Step 3 Assessment Exam

23. A 59-year-old woman comes to the clinic complaining of 24. A 29-year-old man comes to the clinic complaining of a
pain in her hands. She states that her fingers are often burning in his chest. For the past year the patient has
numb and tingling, particularly along her thumb and first had substernal, burning pain in his chest that is usually
three fingers. The pain sometimes wakes her from her present when going to bed. The pain is rated a 5 out of
sleep and requires her to shake her hands until normal 10 on a pain scale and does not radiate, nor is it associ-
sensation returns. The pain, described as sharp tingling ated with any shortness of breath, nausea, or arm and
preceded by numbness, is often worse after knitting, a neck pain. The pain is exacerbated by large meals and
hobby she pursues with great vigor. Aside from this pain, alcohol and is sometimes associated with burping and a
she complains only of mild fatigue and being overweight. foul taste in the back of his mouth. Aside from these
She denies any fevers, chills, visual changes, polyphagia/ symptoms, the man is healthy without any underlying
polydipsia/polyuria, or other systemic symptoms. Past diseases. A review of symptoms does not reveal any
medical history is remarkable only for depression, irrita- fevers, chills, weight loss, anorexia, dysphagia, or
ble bowel syndrome (constipation dominant), and high odynophagia. Vital signs are temperature of 37.0 C
blood pressure treated with a thiazide diuretic. Physical (98.6 F), blood pressure of 128/88 mm Hg, pulse of
examination shows mild wasting of the bilateral thenar 72/min, and respirations of 16/min. Physical examina-
eminences of her palms. Tapping on her wrist increases tion is unremarkable, as are a routine complete blood
the pain, as does prolonged flexion of her wrists. Given count and set of chemistries. An electrocardiogram and
this patient’s presentation, an appropriate test to order a chest radiograph are within normal limits. Given this
when working up the primary cause of her hand pain is patient’s presentation, the most appropriate next step in
his management is to
(A) antinuclear antibody titer
(B) hemoglobin A1c level (A) obtain a breath urease test for H. pylori
(C) no additional testing necessary (B) recommend an empiric trial of omeprazole
(D) thyroid-stimulating hormone (C) recommend that he avoid spicy foods
(E) urine pregnancy test (D) schedule an elective upper endoscopy
(E) start a course of omeprazole/amoxicillin/
clarithromycin

12
Internal Medicine

25. A 37-year-old woman comes to the clinic complaining 26. A 66-year-old man is rushed to the emergency depart-
of diarrhea. Over the past couple of months she has had ment with massive rectal bleeding that began abruptly
4 to 5 loose, brown bowel movements per day. Noticing and painlessly approximately 1 hour ago with a grossly
her fidgety nature, you ask some pointed questions. The bloody, loose bowel movement. A nasogastric lavage is
patient admits to having an unintentional, though performed, which is negative, and stat laboratory stud-
appreciated, 12-pound weight loss over the last 2 ies are sent. A complete blood count at this time reveals:
months. She also has significant heat intolerance; she
often feels hot and sweaty when other people feel that Hematologic
the temperature is normal. Further, many of her friends Leukocyte count 9000/mm3
have noticed that she is irritable, though she attributes
Hemoglobin 7.8 g/dL
this to her recent inability to get a good night’s sleep.
The woman’s vital signs are temperature of 38.0 C Platelet count 200,000/mm3
(100.4 F), blood pressure of 152/92 mm Hg, pulse of
118/min, and respirations of 16/min. She is restless and A transfusion of packed red blood cells is begun, and
fidgety, has thin, fine hair, and displays a slight tremor the gastroenterologist on call is notified of the case. It
when she extends her hands. Ophthalmic examination takes 3 hours to finally get the gastroenterologist and
is remarkable for a stare and lid-lag and exophthalmos. endoscopy nurse ready for a colonoscopy. During this
Her thyroid is diffusely enlarged and nontender, with- time, the patient continues to bleed and requires addi-
out any palpable nodules. Cardiac examination is tional blood. A colonoscopy is performed, but because
remarkable only for tachycardia. The rest of her exami- of massive, brisk bleeding, the exact source is uniden-
nation, with the exception of hyperactive reflexes, is tifiable, though numerous diverticular outpouchings
unremarkable. Laboratory studies are remarkable for are seen in the colon. Surgery then is contacted. After
an elevated free T4, an undetectable thyroid-stimulating an additional hour wait, surgical recommendations
hormone, and the presence of thyrotropin receptor are to contact interventional radiology. During this
antibodies. The aspect of this patient’s presentation time the patient continues to bleed and requires more
most specific for her diagnosis is transfusions of packed red blood cells and continued
fluid boluses to maintain adequate blood pressure.
(A) diarrhea Interventional radiology agrees to perform an angiog-
(B) exophthalmos raphy with embolization but does not have the neces-
(C) goiter sary staff until the morning. Over the next 5 hours the
patient continues to bleed briskly and continues to
(D) hyperreflexia
require blood transfusions. In the morning the patient
(E) lid lag and stare is brought to the interventional radiology suite, where
he has a 2-minute tonic-clonic seizure. In addition to
embolization, an appropriate treatment for this
patient is
(A) calcium
(B) fresh frozen plasma
(C) lorazepam
(D) phenytoin
(E) potassium

13
USMLE Step 3 Assessment Exam

27. A 60-year-old man is being evaluated in the emergency 28. A 19-year-old African American man is brought to the
department for chest pain. While at the triage desk, he emergency department in extreme pain. He states that
suddenly clutches his chest and falls to the ground. He he has a history of sickle cell anemia and is now having
is rushed to the emergency department, where you are extreme chest pain that is worse when he takes a deep
seeing patients. You immediately order that his airway breath. He also is having pain in his back and left ankle.
be maintained and that one of the nurses start provid- The pain began suddenly over the course of the day and
ing positive pressure ventilation with a bag face mask. has been associated with a low-grade fever. The man
The patient is hooked up to a cardiac monitor, and states that he has had pain like this before, and that it
intravenous access is obtained. Based on the monitor usually requires hydromorphone for adequate pain con-
readings and the patient’s lack of a palpable pulse, you trol. He denies any recent fever, chills, diarrhea, dysuria,
call for the defibrillator. After a series of shocks, the fatigue, or other symptoms. His only outpatient medica-
patient is still in the same pulseless rhythm, and CPR is tions are a combination of oxycodone/acetaminophen
resumed. At this point, the pharmacist providing ACLS and a daily multivitamin. Physical examination reveals a
medications suggests that magnesium may be particu- young man in acute distress. Vital signs are temperature
larly useful for this patient’s rhythm. This patient’s of 38.0 C (100.4 F), blood pressure of 168/90 mm Hg,
rhythm, and the rhythm that is most likely to respond pulse of 118/min, and respirations of 26/min. Physical
to magnesium, is examination is remarkable for pallor, a systolic flow
murmur, and mild chest wall tenderness. The rest of the
(A) junctional tachycardia
examination is unremarkable. Laboratory studies fail to
(B) monomorphic ventricular tachycardia reveal any evidence of infection, and the patient
(C) polymorphic ventricular tachycardia improves with hydration, oxygen, and aggressive pain
(D) pulseless electrical activity management. When planning this patient’s discharge, a
treatment likely to reduce recurrences of the events pre-
(E) ventricular fibrillation
cipitating this hospital admission is
(A) exchange transfusion
(B) folate supplementation
(C) home oxygen
(D) hydroxyurea
(E) transdermal fentanyl

14
Internal Medicine

29. A 52-year-old man comes to the emergency department 30. A 66-year-old woman comes to the clinic complaining
complaining of exquisite back pain that started earlier of fatigue and back pain that have become progressively
in the morning. On further questioning, it seems that worse over the past year. A quick review of her chart
the pain is actually along the left side of his abdomen reveals that she suffered a fracture of her left tibia 2
and flank, and radiates toward his groin. The pain is months ago when stepping off a curb. Her laboratory
described as sharp and occurring in waves and can last studies are remarkable for a mildly elevated calcium and
for bursts of up to half an hour. He denies ever having a borderline elevated creatinine level, apparently missed
pain like this in the past. The only associated symptom by the surgeons who treated her fracture. Suspicious of
the man reports is some dark-colored urine. Past med- a serious underlying condition, you order serum and
ical history is significant for diabetes mellitus type 2, urine protein electrophoresis and a beta-microglobulin
complicated by diabetic nephropathy (baseline creati- level, and arrange for a bone marrow biopsy. Serum
nine of 1.9 mg/dL), morbid obesity, and high blood electrophoresis is normal, but urine electrophoresis
pressure, controlled with an ACE inhibitor. Vital signs reveals the presence of Bence-Jones protein. A beta-
are: temperature 38.0 C (100.4 F), blood pressure microglobulin level is elevated, and the bone marrow
150/88 mm Hg, pulse 110/min, and respirations smear shows a marrow composed of 20% plasma cells.
22/min. Physical examination reveals an obviously The patient is understandably distraught when you tell
uncomfortable man who cannot seem to find a com- her the diagnosis. Her major concern in the short term
fortable position on the examination table. Cardiac and is preventing repeat fractures. The most appropriate
pulmonary examinations are normal, and his abdomen next test to look for potential pathologic lesions is
is nontender. Although he states that his flank does
(A) computed tomography (CT) scan
indeed hurt, the pain is not worse with palpation. A
dipstick urinalysis reveals a large amount of blood with (B) full body roentgenography (x-ray)
no white blood cells, leukocyte esterase, or nitrates pre- (C) magnetic resonance imaging (MRI)
sent. Given this patient’s likely condition, the most (D) positron emission tomography (PET) scan
appropriate and sensitive confirmatory diagnostic test
(E) technetium-99m uptake-scan (bone scan)
to order is a/an
(A) abdominal and renal ultrasound
(B) contrast enhanced CT scan of the abdomen/pelvis
(C) intravenous pyelogram (IVP)
(D) KUB (kidney/ureters/bladder) radiograph
(E) noncontrast spiral CT scan of abdomen/pelvis

15
USMLE Step 3 Assessment Exam

31. A 48-year-old woman arrives in the emergency depart- 32. A 68-year-old man is brought to the emergency depart-
ment at 3 AM complaining of moderate to severe right ment in respiratory distress. The man has a long history
upper quadrant abdominal pain, fevers, sweats, and of tobacco use and was recently diagnosed with chronic
chills. She has had intermittent right upper quadrant bronchitis following pulmonary function tests that
pain over the last 2 months, but that pain has been brief revealed an obstructive ventilatory defect. Between pres-
and intermittent and is described as dull and crampy and sured breaths, the man reports having a recent increase
often exacerbated by meals. Over the past day, however, in the production of sputum, which now appears thick
the pain has become more intense and unremitting and and green, and some low-grade fevers over the past few
feels as if it radiates through toward her upper back. days. He is tachypneic and cyanotic and requires emer-
Directed questioning reveals a history of dark-colored gency intubation. Once stabilized and transferred to the
urine and light-colored stools. Physical examination intensive care unit, the patient is treated with intravenous
reveals a moderately overweight woman in obvious dis- steroids, antibiotics, and nebulized albuterol and iprat-
comfort. Her temperature is 38.4 C (101.1 F), blood ropium. The patient slowly improves and is eventually
pressure is 100/70 mm Hg, pulse is 105/min, and respi- extubated. Once stable, an arterial blood gas reveals:
rations are 16/min. Examination is remarkable for mild-
ly icteric sclera and a positive Murphy’s sign. Laboratory pH 7.35
studies are remarkable for a leukocytosis and a liver func- PCO2 60 mm Hg
tion panel revealing a cholestatic pattern. A right upper
PO2 52 mm Hg
quadrant ultrasound is ordered and is pending. Given
this patient’s presentation, the most important next step
This arterial blood gas is similar to one drawn in the
in her management is to
clinic approximately 2 weeks before his exacerbation.
(A) admit to medicine service and begin ticarcillin/ Laboratory studies are also remarkable for an elevated
clavulanic acid bicarbonate level and a normocytic anemia. In addition
(B) arrange for emergent endoscopic ultrasound to smoking cessation, the outpatient treatment most
likely to prolong this patient’s life is
(C) call GI fellow for endoscopic retrograde cholan-
giopancreatography (A) albuterol metered dose inhaler
(D) call surgery to place T-tube for biliary decompres- (B) continuous home oxygen
sion (C) ipratropium metered dose inhaler
(E) send patient to the operating room for exploratory (D) pulmonary rehabilitation
laparotomy
(E) steroid metered dose inhaler

16
Internal Medicine

33. A 72-year-old woman comes to the clinic complaining 34. A 65-year-old man comes to the emergency department
of a severe unilateral right-sided headache. The pain, after vomiting bright red blood twice at home approxi-
which she currently states is a throbbing 7/10, has built mately 4 hours ago. He has also had stomach pain for
up over the last 2 weeks, during which she has had near- the last 2 weeks. He denies any other symptoms and
daily headaches. The headaches have been partially specifically denies feeling dizzy, having any palpitations
relieved with acetaminophen, but given that they seem or confusion, or having any chest pain. He also denies
to be intensifying, she finally decided to seek help. In weight loss, fatigue, anorexia, or persistent nausea. The
addition to her headaches, she has had blurred vision patient reports having severe osteoarthritis, for which
out of her right eye for the past day and has been run- he has been taking high-dose indomethacin. He also
ning a low-grade fever. She has never had symptoms sheepishly admits to drinking “a fair bit” over the last
such as these before, but does have a past medical his- weeks, as there have been numerous holiday parties.
tory that includes hypertension, hypercholesterolemia, Vital signs are: temperature 37.0 C (98.6 F), blood pres-
and polymyalgia rheumatica that recently has become sure 140/90 mm Hg, pulse 90/min, and respirations
worse. Vital signs are temperature 38.0 C (100.4 F), 20/min. Examination reveals some mild epigastric pain
blood pressure 170/90 mm Hg, pulse 90/min, and res- on palpation, but is generally unremarkable. Stat blood
pirations 18/min. Examination is remarkable for signif- work reveals a hematocrit of 35%, compared with the
icant tenderness anterior to the patient’s right tragus patient’s baseline hematocrit of 41%. A nasogastric
and a visual acuity of 20/30 OS and 20/200 OD without lavage is performed that reveals a large amount of dark,
any photophobia. She has an erythrocyte sedimentation coffee-ground material but no active bleeding. The
rate of 100 mm/h and a normocytic anemia. Given this patient receives an intravenous H2-blocker and, the
patient’s therapy, the most appropriate next step in the next morning, an upper endoscopy. Endoscopy without
management of her condition is biopsy given the risk for bleeding shows a large gastric
ulcer and some diffuse gastritis. The ulcer seems to have
(A) broad-spectrum antibiotics
a clean base and distinct margins. Helicobacter pylori
(B) high-dose steroids testing is negative. In addition to acid suppression ther-
(C) lumbar puncture with CSF analysis apy, an appropriate followup test for this patient is
(D) MRI of the brain (A) barium swallow study
(E) refer for a temporal artery biopsy (B) CT scan of chest, abdomen, and pelvis
(C) no further testing, follow patient clinically
(D) repeat H. pylori testing in 6 months
(E) repeat upper endoscopy and biopsy

17
USMLE Step 3 Assessment Exam

35. A 63-year-old man comes to the emergency department 36. A 28-year-old woman comes to the clinic complaining
complaining of shortness of breath and chest pain. The of “tightness” in her hands. She states that the skin of
symptoms began abruptly, just as he was pulling some her hands feels taut, and that over the last 6 to 8 months
baggage off a Greyhound bus. He had just returned she has developed small, slowly healing ulcerations on
from visiting his son, a good 2-day bus ride away. His her fingertips. Before this she had suffered severe
pain is worse with inspiration, and he has had two Raynaud phenomena, but has never had ulcerations. A
episodes of coughing spells, resulting in blood-streaked careful review of systems reveals that the patient also
sputum. Further, over the course of the day the patient suffers from severe GERD symptoms, has a history of
has felt his heart racing, and he has developed a low- recently diagnosed hypertension, and recently has
grade fever. He has a history of lung disease, having noticed severe shortness of breath with exertion. Vital
been diagnosed with silicosis 2 years ago after a workup signs reveal a temperature of 37.0 C (98.6 C), blood
for pulmonary nodules and a chronic bronchitis. pressure of 155/95 mm Hg, pulse of 73/min, and respi-
However, he has never suffered symptoms similar to the rations of 18/min. Physical examination confirms the
ones he currently has. Vital signs are temperature of presence of taut, sclerotic skin, occasional facial telang-
38.3 C (101.0 F), blood pressure of 158/98 mm Hg, iectasias, and two small ulcerations at the tips of the
pulse of 110/min, and respirations of 28/min. Cardiac second and third digits of the right hand. Her fingers
and lung auscultations are unremarkable. An EKG are thin and spider-like. Chest examination reveals
reveals sinus tachycardia with a right axis deviation. good airflow. Cardiac auscultation reveals a widely split
The patient has no lower extremity edema, palpable S2, but is otherwise unremarkable. An autoimmune
chords, and displays a negative Homans sign. The most antibody panel reveals the following:
appropriate test to diagnose this patient’s acute condi-
tion is Serologies
(A) chest radiograph Rheumatoid factor Positive
(B) CT angiography Antinuclear antibodies Positive
(C) D-dimer level Anticentromere antibodies Negative
(D) pulmonary angiography Anti-SCL-70 (topoisomerase I) Positive
(E) ventilation/perfusion scan
A complete blood count is remarkable for a normocytic
anemia with a hematocrit of 29%, and a set of
chemistries reveals a serum creatinine of 1.6 mg/dL.
Urinalysis reveals 2+ proteinuria. Given this patient’s
presentation, the most appropriate medication to start is
(A) hydrochlorothiazide
(B) ibuprofen
(C) lisinopril
(D) methotrexate
(E) prednisone

18
Internal Medicine

37. A 42-year-old man comes to the clinic complaining of a 38. A 70-year-old man comes to the emergency department
lump in his neck. He noticed the neck swelling maybe a complaining of substernal chest pain radiating to his left
month ago, but has not thought much of it, as he has arm. The pain occurred earlier in the day while he was
been too busy to come see a doctor. Because the lump mowing his lawn and has since resolved. It was not asso-
seems slightly larger, however, he is worried. The man ciated with nausea, shortness of breath, or diaphoresis,
reports having Hodgkin lymphoma as a child, which and resolved with rest. The patient has had pain like this
required radiation therapy to his chest and neck, and he before and states that it usually resolves with rest or a
is worried that it may have recurred. He denies any dys- sublingual nitroglycerin. Unfortunately, he recently ran
phagia, odynophagia, hoarseness, or signs and symptoms out of his medications and his concerned neighbor
of hyperthyroidism. Family history is unremarkable for insisted that he come to the emergency department. His
any thyroid, head and neck, or other malignancies. Vital current medications include a beta-blocker, an ACE
signs are within normal limits. Palpation of the thyroid inhibitor, an over-the-counter acetaminophen for pain,
reveals an enlarged, nontender gland with a large, firm a daily long-acting nitroglycerin pill, and sublingual
nodule in the lower pole of the right lobe. There is no nitroglycerin as needed. Vital signs are: temperature 37.0
associated lymphadenopathy, and the rest of the exami- C (98.6 F), blood pressure 132/80 mm Hg, pulse 68/min,
nation is unremarkable. A chest radiograph obtained in and respirations 16/min. Cardiac examination reveals a
the office reveals calcification of the thyroid gland. displaced, prominent point of maximal impulse and a
Thyroid studies are pending, and a fine needle aspiration fourth heart sound. There is no jugular venous disten-
is scheduled for later in the week. If this patient’s nodule tion, peripheral edema, or murmurs, rubs, or gallops.
is malignant, the most likely diagnosis is The rest of the examination is normal. An electrocardio-
gram shows large voltages in the precordial QRS com-
(A) anaplastic carcinoma
plexes, but no ST or T wave changes. The most appro-
(B) follicular carcinoma priate course of management is to
(C) medullary carcinoma
(A) admit to cardiology for urgent cardiac catheteriza-
(D) papillary carcinoma tion
(E) thyroid lymphoma (B) discharge with refills for 1 month and recommend
daily aspirin
(C) draw a set of cardiac enzymes, admit for monitor-
ing of serial enzymes
(D) order a stress echocardiogram or nuclear perfu-
sion study
(E) order a stress electrocardiogram as an outpatient

19
USMLE Step 3 Assessment Exam

39. An 84-year-old man is brought to the emergency 40. A 60-year-old man comes to the clinic complaining of
department after having an episode of hematemesis at a itchiness. After a warm bath or shower or after vigorous
skilled nursing facility. According to the nursing home rubbing of his skin, he suffers from severe pruritus, which
staff, the patient has not been feeling well for the past is relieved only partially with antihistamines (topical and
week and, although he has taken his medications as pre- systemic), but seems to respond well to aspirin. An exten-
scribed, he has not been eating or drinking much food sive review of symptoms reveals vague complaints of
or fluids. The patient had some diarrhea that has intermittent headaches and rare dizziness, but is other-
tapered off over the past few days, though some other wise unremarkable. Past medical history is significant for
members of the nursing home have had similar prob- tobacco use, stage one hypertension, and obstructive sleep
lems. The patient’s past medical history is significant apnea, treated 1 year ago with a surgical procedure to
for osteoarthritis, hypertension, and atrial fibrillation. reduce upper airway occlusion. At the time of the opera-
Current medications, some prescribed by the nursing tion, the patient was told he had “high red blood” and to
home physician and some by the patient’s previous follow up with his primary care physician, which he did
primary care provider, include acetaminophen, ibupro- not do. The man denies any fevers, chills, night sweats,
fen, hydrochlorothiazide, warfarin, and a recent course changes in weight or appetite, fatigue, recent infections, or
of ciprofloxacin for gastroenteritis. Vital signs are tem- other symptoms. Examination is remarkable for facial
perature of 37.0 C (98.6 F), blood pressure of 130/80 plethora and a palpable spleen 2 cm below the left costal
mm Hg, pulse of 110/min, and respirations of 16/min. margin. Reviewing the patient’s medical records, you find
Physical examination is remarkable for multiple ecchy- a preoperative hematocrit level of 60% and suspect poly-
motic patches and some mild abdominal tenderness, cythemia vera. If this is indeed the correct diagnosis, an
but is otherwise normal. Laboratory studies show: associated expected laboratory finding is
(A) low blood viscosity and red cell mass
Hematologic
(B) low leukocyte alkaline phosphatase level
Hematocrit 36%
(C) low serum cyanocobalamin level
Leukocyte count 9000/mm3
(D) low serum erythropoietin level
Neutrophils 64%
(E) low white blood cell and platelet counts
Lymphocytes 32%
Platelets 110,000/mm3
Prothrombin time/INR 120 s/10
Partial thromboplastin time 29 s

Given this patient’s presentation, the most appropriate


initial treatment for his bleeding disorder is
(A) cryoprecipitate transfusion
(B) DDAVP
(C) platelet transfusion
(D) protamine
(E) vitamin K

20
Internal Medicine

41. A 70-year-old woman is brought from a nursing home 42. A 72-year-old man is brought to the emergency
after having a witnessed grand mal seizure at her department after being found to have an altered state
daughter’s home, where the patient currently resides. of consciousness. Over the past week he had become
Apparently, she has had increased confusion over the increasingly confused at his nursing home, though the
past day and has developed a low-grade fever over the nurse on duty reported that he has had normal vital
past few hours. She is now acutely disoriented and con- signs and has been afebrile. Aside from a chronic
fused. Past medical history is remarkable only for cough, he had apparently been in good health over the
hypertension and polymyalgia rheumatica, and current last few months. Physical examination reveals a mildly
medications include aspirin, a beta-blocker, and a pred- disoriented man with some crackles and bronchial
nisone taper. Her temperature is 38.0 C (100.4 F), blood breath sounds over his right middle lobe, but an other-
pressure is 110/80 mm Hg, pulse is 78/min, and respi- wise unremarkable examination. Initial chemistries are
rations are 20/min. Ophthalmoscopic examination is remarkable for a sodium level of 121 mEq/L. A chest
normal, without any papilledema, though mild photo- radiograph reveals a large right-sided mass with indis-
phobia is present. Head and neck examination reveals tinct borders concerning for malignancy. Additional
mild nuchal rigidity. The patient knows her own name, studies are ordered to evaluate the patient’s hypona-
but is uncertain where she is or what the date is. There tremia, and are as follows:
are no focal motor or sensory deficits, and toes are
downgoing bilaterally. Empiric ceftriaxone is started in Urine sodium 70 mmol/L
the emergency department, and the patient receives a Urine osmolality 500 mOsmol/kg H2O
stat CT scan of the head, which is read as normal, with-
Serum osmolality 250 mOsmol/kg H2O
out evidence of increased intracerebral pressure. A lum-
bar puncture then is performed, and the patient is sent
The patient is admitted to the medicine floor for fur-
to the medicine floor. Results of the Gram stain reveal a
ther evaluation and treatment. Given this presentation,
predominantly neutrophilic leukocytosis, an extremely
the most appropriate initial treatment for his hypona-
low glucose relative to serum glucose, and an elevated
tremia is
protein. Gram stain shows small gram-positive rods
and coccobacilli. The patient is continued on empiric (A) demeclocycline
ceftriaxone. Twenty-four hours after admission to the (B) fluid restriction
floor, she has another seizure that, despite appropriate
(C) hydrochlorothiazide
emergent treatment, results in a compromised airway
and subsequent respiratory arrest. An additional appro- (D) hypertonic saline
priate treatment that may have prevented this patient’s (E) normal saline
condition is the empiric addition of
(A) acyclovir
(B) ampicillin
(C) dexamethasone
(D) piperacillin/tazobactam
(E) vancomycin

21
USMLE Step 3 Assessment Exam

43. A 30-year-old African American woman comes to the 44. A 70-year-old woman is being evaluated for a new-
clinic complaining of a chronic cough. Additional com- onset dementia. Over the past year she has suffered
plaints include aching joints and a strange rash on her from increasingly severe cognitive deficits and has sig-
right tibia. These symptoms have bothered her for nificant difficulty with forming new memories, remem-
approximately 2 months and seem to be getting worse. bering the names of children and grandchildren, and
Furthermore, over the last week she has suffered from can barely concentrate on simple tasks. Additionally,
red eyes and occasional blurred vision. The cough is her family reports that she now requires adult diapers
worse with exertion, and she frequently needs to stop to and has developed a clumsy walk and often falls.
catch her breath. She denies any tobacco history, asth- Physical examination reveals a distracted, distant
ma, recent travel, or relevant occupational exposures woman in no apparent distress. She has a bradykinetic,
and has not had similar symptoms in the past. The wide-based, shuffling gait, but does not have true trun-
woman has not suffered any recent fevers, chills, recur- cal ataxia or weakness. She speaks in few words, but is
rent infections, weight loss, night sweats, or other con- alert and oriented to her surroundings and agrees that
stitutional symptoms. Past laboratory studies have con- she is not as sprightly as she once was. The mini-men-
firmed that the patient is HIV-negative and, 1 month tal status examination confirms deficits in near-term
earlier, PPD-negative. Physical examination is remark- memory, concentration, and executive functioning. The
able for some faint bilobar inspiratory crackles. Her leg rest of her examination is normal. Recent laboratory
has multiple, tender, violaceous, subcutaneous nodules. studies have included normal thyroid studies, a normal
Further laboratory workup reveals an elevated calcium vitamin B12 level, a negative syphilis screening test, and
level, an elevated erythrocyte sedimentation level, and a normal complete blood count. A hyponatremia of 132
an elevated serum angiotensin level. Bilateral hilar mEq/L was noted on a set of chemistries, but laborato-
adenopathy is seen on a chest radiograph, together with ry studies have been otherwise unrevealing. A CT scan
some basilar consolidation. Pulmonary function testing reveals massive ventricular enlargement without signif-
with spirometry and carbon monoxide diffusion capac- icant sulcal atrophy. Based on this patient’s presenta-
ity (DLCO) testing shows a mild restrictive defect with tion, the most appropriate management is to
reduction in the DLCO. Based on this patient’s presen-
(A) begin treatment with carbidopa/levodopa
tation, an expected finding with further workup is
(B) begin treatment with vitamin E and donepezil
(A) acid-fast bacillus seen on induced sputum smear
(C) explain that no further treatment is available
(B) elevated serum levels of 1,25-dihydroxy cholecal-
(D) refer for CSF shunt procedure
ciferol
(E) slowly replete sodium to normal
(C) lymph node biopsy showing granulomas with cen-
tral necrosis
(D) prominent and severe reduction in CD4 cell count
(E) skin biopsy of rash that reveals noncaseating gran-
ulomas

22
Internal Medicine

45. A 48-year-old man is brought to the emergency depart- 47. A 24-year-old woman comes to the clinic complaining
ment after being found confused and incoherent in the of joint pain. For the last couple of months, she has had
park. Apparently, the patient has a long history of intermittent, diffuse joint pain, mainly in her hands and
admissions for altered mental status and a history of wrists. Over the last month, she also has noticed some
end-stage alcoholic cirrhosis. In the hospital he is som- swelling in her knuckles. Although her pain is con-
nolent but arousable. He is disoriented and does not trolled with over-the-counter acetaminophen and
know the date or his current location. Vital signs are: aspirin, she is worried that something more serious
temperature 37.0 C (98.6 F), blood pressure 104/88 mm may be occurring, as she has lost 20 pounds over the
Hg, pulse of 74/min, and respirations18/min. Chest is past month, has had intermittent fevers, and feels tired
clear to auscultation, and cardiac examination reveals all the time. Past medical history is significant for a
normal heart sounds without any murmurs, rubs, or “nervous breakdown” approximately 6 months ago
gallops. Percussion and palpation of the abdomen requiring a short hospital stay. Physical examination is
reveal splenomegaly, though no hepatomegaly. Shifting remarkable for a scaly, circular rash with a raised rim
dullness is present, as is a visible fluid wave. and some central necrotic areas that spread over the
Examination of the skin shows multiple telangiectasias, scalp and ears. Additionally, two small, shallow, pin-
rhinophyma, and palmar erythema. Laboratory studies point ulcers with clean bases are seen on the inside of
show an elevated PT/INR, a decreased protein and her lower lip. Examination of her digits reveals mild
albumin, and a normal white blood cell count. A serum warmth and erythema of the proximal interphalangeal
venous ammonia level is elevated, and a urinalysis and (PIP) and metacarpal (MC) joints. Laboratory studies
a chest radiograph are normal. The patient is given lac- are remarkable for a positive ANA, a normocytic ane-
tulose by way of a nasogastric tube, and shortly after mia, positive anti-Smith and dsDNA antibodies, posi-
having two large bowel movements seems more awake tive anti-RNP antibodies, and low complement levels.
and oriented. The most sensitive finding that would The most specific finding characteristic of this patient’s
support this patient’s likely diagnosis is condition is
(A) asterixis (A) her rash
(B) fetor hepaticus (B) low complement levels
(C) flaccidity (C) positive ANA
(D) hyporeflexia (D) positive anti-RNP antibodies
(E) nystagmus (E) positive anti-Smith antibodies

46. A 49-year-old woman comes to the clinic complaining


of fatigue and weight gain. Over the past 6 months she
has felt as if she has no energy and has little interest in
doing much of anything. She does not feel that her
mood is down, but rather she seems to lack the energy
and volition to do much. Further, she has gained 10
pounds without any change in her diet. She feels cold
almost all the time and suffers from significant consti-
pation. Physical examination is remarkable for a visible
nontender goiter, dry, doughy skin, and deep tendon
reflexes with a delayed relaxation phase. The most
appropriate and first laboratory study to order to diag-
nose and follow this patient’s condition is
(A) free thyroxine (T4) index
(B) thyroid-stimulating hormone (TSH)
(C) thyroxine (T4), free
(D) thyroxine (T4), total
(E) triiodothyronine (T3), free

23
USMLE Step 3 Assessment Exam

48. A 40-year-old woman comes to the emergency depart- 49. A 68-year-old man comes to the clinic complaining of
ment complaining of palpitations. Further questioning shortness of breath and difficulty sleeping. Apparently,
reveals that these palpitations, which began last night, when he lies down he often feels as if he is drowning and
also are associated with significant shortness of breath, will sometimes wake at night gasping for air. He also
mild pleuritic pain, and a cough. She is a chain smoker, often wakes at night needing to use the bathroom, a
though she knows she has to quit, but has no chronic problem he did not suffer from until recently. When
illnesses or history of cardiopulmonary disease. Her walking or even climbing stairs, he is quickly winded
only medication is an oral contraceptive pill. Vital signs and needs to rest to catch his breath. Further question-
are temperature of 38.0 C (100.4 F), blood pressure of ing reveals a chronic cough, occasionally productive of
134/88 mm Hg, pulse of 120/min, and respirations of pink, frothy sputum, lower extremity swelling, and a 15-
26/min. Chest auscultation reveals some mild inspira- pound weight gain over the past few months. His past
tory crackles and obvious splinting. The rest of the medical history is significant for two myocardial infarc-
examination is unremarkable. An electrocardiogram tions, a history of hypertension, and borderline elevated
reveals sinus tachycardia, with the presence of a large S- fasting blood glucose levels. The patient currently takes
wave in lead I, a small Q wave in lead III, and a flipped a beta-blocker and a daily aspirin. Vital signs are: tem-
T-wave in lead III. The patient is sent for a ventila- perature of 37.0 C (98.6 F), blood pressure 105/70 mm
tion/perfusion scan that reveals a large area of ventila- Hg, pulse 110/min, and respirations 22/min. Physical
tion without perfusion in her right lung. The patient is examination reveals bibasilar rales in his chest, a 2/6 sys-
started on a heparin drip and is admitted to the tolic murmur heard over the apex, and a third heart
medicine service. Based on this, the most appropriate sound. Jugular venous pressure is approximately 10 cm
anticoagulation for this patient is H2O, and the lower extremities display 2+ pitting
edema. The rest of the examination is normal. As part of
(A) daily aspirin or clopidogrel
the patient’s workup, he receives an echocardiogram
(B) placement of a Greenfield filter that reveals anterior wall motion abnormalities consis-
(C) 6 months of heparin tent with his previous myocardial infarction. The esti-
(D) 3 to 6 months of enoxaparin mated left ventricular ejection fraction is 20 to 25%.
Given this patient’s condition, he is started on treatment
(E) 3 to 6 months of warfarin
for congestive heart failure. Among the agents available
to treat this patient’s condition, the drug most likely to
improve his chances of survival is
(A) digoxin
(B) furosemide
(C) hydrochlorothiazide
(D) spironolactone
(E) terazosin

24
Internal Medicine

50. A 68-year-old woman is referred to the clinic for a 52. A 48-year-old man comes to the emergency department
preoperative evaluation before a left breast lumpectomy. complaining of abdominal distention. He has a history
On a routine presurgical evaluation, she was found to of hepatitis C-related cirrhosis and is currently on the
have an isolated prolonged partial thromboplastin time, liver transplant waiting list. His cirrhosis has resulted in
and the surgeon wishes to make sure that, if intraopera- edema and mild ascites, which have been controlled
tive frozen sections warrant a more expansive operation, with spironolactone and a low-sodium diet. Over the
the patient is not at risk for major hemorrhage. The past week, however, the man has noticed increasing
woman reports feeling well and, aside from a distant his- abdominal girth, despite continued use of his medica-
tory of syphilis that she says was picked up by a labora- tions. His wife, who has accompanied the patient, also
tory study that was part of a military employment exam- notes that he has seemed more confused and has had
ination, she has been healthy. She has no major illnesses, occasional low-grade fevers. The patient denies any
takes no medications, and can think of no illnesses that chills, headaches, nausea, severe abdominal pain,
affect herself, her parents, or her four healthy daughters. change in stool color or consistency, cough, or dysuria.
Factor mixing studies are performed, but the patient’s Vital signs are: temperature 38.0 C (100.4 F), blood
coagulation parameters do not improve. Although an pressure 108/80 mm Hg, pulse 90/min, and respirations
ANA and an FTA-ABS are negative, a Russell viper 20/min. Physical examination is remarkable for scat-
venom test is positive. Based on this presentation, the tered telangiectasias, palmar erythema, and mild
most appropriate treatment for this patient is splenomegaly. The patient’s abdomen displays bulging
flanks, shifting dullness, and a positive fluid wave. The
(A) daily vitamin K supplementation
abdomen is tender but soft, and there is no guarding or
(B) fresh frozen plasma before surgery rebound tenderness. Abdominal sounds are normal.
(C) no further treatment Laboratory studies show:
(D) warfarin, goal INR of 2 to 3 seconds
Hematologic
(E) warfarin, goal INR of 2.5 to 3.5 seconds
Leukocytes 9000/mm3
Hematocrit 31%
51. A 56-year-old African American man comes to the office Platelets 78,000/mm3
for an annual health care checkup. As part of routine
PT 20 seconds
healthcare maintenance, you offer the patient prostate
cancer screening, as his ethnicity and family history (an PTT 28 seconds
uncle with prostate cancer) place him at risk for the dis-
ease. The man denies any current problems and has had A chest radiograph reveals a small, right-sided effusion,
no weight loss, bony pain, change in appetite or weight, but no infiltrates, and a KUB film is essentially normal.
or any other systemic symptoms. He denies any urinary A therapeutic paracentesis 1 month ago was normal.
frequency, dysuria, nocturia, incomplete voiding, or Based on this patient’s presentation, the most appropri-
weak urinary stream. Digital rectal examination, howev- ate next step in the management of his condition is to
er, reveals a firm, asymmetric prostate with a small, left- (A) add furosemide to medication regimen
sided nodule. A prostate-specific antigen (PSA) test is
(B) begin empiric cefotaxime immediately
ordered, the results of which should be available some-
time next week. In addition to a PSA test, an additional (C) order abdominal CT scan and/or ultrasound
appropriate test to order at this time is (D) perform diagnostic paracentesis
(A) CT scan of abdomen and pelvis (E) perform diagnostic thoracentesis
(B) no additional tests at this time
(C) radionucleotide bone scan
(D) surgical biopsy
(E) transrectal ultrasound

25
USMLE Step 3 Assessment Exam

53. A 45-year-old woman comes to the urgent care clinic 54. As an upcoming intern, your hospital requires you to
complaining of abdominal pain. For the last 2 days she take a host of employment-related examinations. In
has had pain in her lower abdomen and right flank, addition to hours of standardized testing, you need to
and has felt weak and feverish. She reports diffuse prove prior immunization status, have a general physi-
severe nausea and vomiting, fatigue, and increased uri- cal examination, and have a PPD test for tuberculosis.
nary frequency. The woman denies any recent sexual Your immunizations are up to date, and you pass your
contacts, but notes that she has had chlamydia in the physical examination with flying colors. The test for
past, which was treated successfully 2 years ago. Vital tuberculosis exposure, however, makes you anxious.
signs are: temperature 39.2 C (102.5 F), blood pressure Although you are not immunocompromised and do
138/90 mm Hg, pulse 108/min, and respirations not have any chronic diseases, travel exposures, or
18/min. Examination is remarkable for suprapubic known TB contacts, you have heard of unsuspecting
tenderness and right costovertebral angle tenderness. interns being subjected to a prolonged course of INH
The rest of the examination, including bimanual pelvic therapy. The PPD is placed, and you are told to come
examination, is unrevealing. Laboratory studies show: back in 2 days. Given this scenario, a positive PPD
would be
Urinalysis
(A) an area of erythema greater than 5 mm
3+ leukocyte esterase; 3+ nitrate; many WBCs
(B) an area of erythema greater than 10 mm
and bacteria; trace protein
(C) an area of induration greater than 5 mm
Gram stain
(D) an area of induration greater than 10 mm
Many gram-negative bacilli (urine)
(E) both erythema and induration greater than 15 mm
Urine culture
Results pending
CBC and differential 55. A 62-year-old man is brought to the clinic by his
Results pending concerned spouse. Over the past 6 months, the woman
tells you that her husband has not quite been himself.
On the basis of these results, the woman is admitted to He often stares into space, barely speaks, and often
the medicine service for intravenous antibiotics. An seems slow to respond to questions. She has noticed
additional appropriate confirmatory test to order at this that he tries not to walk, because when he does he often
time is a(n) seems to fall. As a result, she tells you, he walks slowly
and with a slow, shuffling gait. Physical examination
(A) abdominal and renal ultrasound reveals a well appearing but distracted elderly gentle-
(B) CT scan of abdomen and pelvis man. He has little facial expression and takes a long
(C) intravenous pyelogram (IVP) time to answer questions, though he is appropriate
when he finally does speak. His movements are
(D) KUB radiograph
markedly bradykinetic and he has an obvious tremor
(E) no additional tests that improves with focused movement. Recent labora-
tory studies are unremarkable. Based on this presenta-
tion, the treatment most likely to delay the progression
of this patient’s condition is
(A) amantadine
(B) carbidopa/levodopa
(C) donepezil
(D) sertraline
(E) vitamin B12 and folate

26
Internal Medicine

56. A 56-year-old man awaiting a liver transplant comes to 57. A 48-year-old woman with long-standing diabetes mel-
the clinic complaining of increased abdominal girth. litus comes to the office for a routine checkup. She is
He suffers from hepatitis C cirrhosis complicated by a extremely compliant and has a good understanding of
variceal bleed, portal gastropathy, and mild to moderate the disease. Her regimen includes a mix of NPH insulin
ascites. This ascites has gotten worse over the last 1-2 taken twice a day and regular insulin taken before
weeks. The man’s pants can no longer be buttoned and meals. She is concerned, as she has noticed that her
he has gained 12 pounds. He denies any fevers, chills, or blood sugar levels recently have been less controlled
abdominal pain and has not been more confused than than normal. Although she admits that her diet has
usual. He has not changed his diet and is meticulous changed somewhat since she started taking Thai
about taking his medications regularly, which include cooking classes, she feels she has adjusted her insulin
furosemide and spironolactone. Vital signs are unre- appropriately. Her only other complaints, aside from
markable, and physical examination, aside from an morning hyperglycemia, are fatigue and poor sleep.
obvious fluid wave and pronounced shifting dullness, is Over the past few weeks she has had numerous episodes
unchanged from previous examinations. A diagnostic in which she wakes early in the morning or late at night
paracentesis reveals the presence of a small number of with shakes, chills, and occasional sweats. The symp-
leukocytes (100/mm) and no organisms on Gram stain. toms resolve with drinking some orange juice or taking
An ultrasound is then performed that, because of the a small snack, and she feels better in the morning,
man’s girth, is technically difficult. The liver is visual- though her blood sugar often is then elevated at 200 to
ized and seems to have no clot in the hepatic veins or 300 mg/dL. Given this patient’s presentation, the most
visualized portion of the inferior vena cava. The liver effective strategy for controlling her blood glucose lev-
appears nodular and small and seems to have a 3-cm els and nocturnal symptoms is to
mass in the right lobe. The most appropriate addition-
(A) decrease the dose of evening NPH insulin
al laboratory study to order to determine if this mass
represents a hepatoma is (B) decrease the dose of regular insulin before dinner
(C) encourage the patient not to have nocturnal snacks
(A) alpha-fetoprotein (AFP)
(D) increase the dose of morning NPH insulin
(B) beta-human chorionic gonadotropin (β-hCG)
(E) increase the dose of regular insulin before breakfast
(C) cancer antigen 125 (CA-125)
(D) carcinoembryonic antigen (CEA)
(E) prostate-specific antigen (PSA)

27
USMLE Step 3 Assessment Exam

58. A 54-year-old woman comes to the emergency depart- 59. A 57-year-old homeless man comes to the emergency
ment complaining of severe substernal chest pain and department complaining of vomiting bright red blood
difficulty breathing for the last 45 minutes. The woman after drinking his usual fifth of vodka. The patient is
has a long list of cardiac risk factors, including tobacco hemodynamically stable, has a stable hematocrit of
use, hypertension, dyslipidemia, and a family history of 38%, and has no bright red blood from nasogastric
coronary heart disease. In addition, she suffers from lavage, though copious amounts of coffee grounds are
severe rheumatoid arthritis, for which she has been tak- present. Hearing this over the phone, the gastroenterol-
ing prednisone, and obstructive sleep apnea. An elec- ogy fellow on call decides to delay upper endoscopy
trocardiogram reveals large ST segment elevations in until the morning. The patient is made NPO and is
leads V1 through V6, and, because a cardiac catheteri- placed on intravenous maintenance fluids, specifically
zation laboratory is not available in the hospital or sur- D51/2NS at 100 mL/h. Early in the morning the patient
rounding area, the patient receives intravenous tissue has three large, maroon, foul-smelling bowel move-
plasminogen activator (tPA). Despite early reperfusion, ments and begins to vomit bright red blood. Over the
she develops cardiogenic shock from a massive anterior next 6 hours, the patient receives 5 L of crystalloid
myocardial infarction, requiring intubation and treat- (D5NS) and 6 U of colloid (4 U of packed red blood
ment with intravenous dobutamine. Over the next 4 cells and 2 U of fresh frozen plasma). He requires intu-
days, the patient is weaned slowly from the dobutamine bation to control his airway and receives a bedside
as her cardiac function improves. When the sedation is upper endoscopy in the intensive care unit. Diffuse gas-
lowered and the patient is planned to be extubated, tritis and a large bleeding peptic ulcer are seen. The
however, she is unable to move her arms or legs and is ulcer is clipped and injected with epinephrine and the
unable to create a high negative inspiratory force dur- bleeding stops. Over the next 2 days, the patient is
ing ventilator weaning trials. She can, however, open weaned from ventilatory support. When the patient is
and close her eyes on command and seems to under- finally extubated, however, he is found to be confused
stand what is said to her. The most likely cause of this and inattentive to examination. He has bilateral lateral
patient’s current symptoms is rectus palsies and significant nystagmus. When physical
therapy attempts to walk the patient, he is unable to
(A) atlantoaxial subluxation
maintain a steady stance and immediately falls over.
(B) cervical epidural hematoma The treatment most likely to have prevented this
(C) hypoxic brain damage patient’s current condition is
(D) intracerebral hemorrhage (A) calcium
(E) long-term prednisone use (B) folate
(C) magnesium
(D) vitamin B1
(E) vitamin B12

28
Internal Medicine

60. A 72-year-old man comes to the office complaining of 61. A 44-year-old African American man comes to the clin-
leg pain. The man is well known to you, as he has a long ic for a routine checkup. Over the past year the man has
history of vascular disease, including two previous suffered from mild arthralgias, intermittent abdominal
myocardial infarctions and a carotid endarterectomy pain, and worsening fatigue. Additionally, he has devel-
following a stroke 2 years ago. Since his stroke the man oped erectile dysfunction, stating simply that he has lost
has regained his ability to walk, after completing a long the volition and ability to have sex. He has no idea why
rehabilitation program. Now when he walks even half a he is suffering from these problems and denies any
block, however, he suffers dull, crampy pain in his left depressive symptoms. Before this, he has been healthy
calf. The pain improves with rest and has never woken and does not smoke, drink, or use any illicit substances.
him from sleep, but it is making it nearly impossible for Aside from a father who died of diabetes, there are no
him to complete his regular daily activities or to com- family medical problems. Examination is remarkable
plete his “heart healthy” exercise routine. Current med- for hepatomegaly and a faint bronze hue to the skin. A
ications include baby aspirin, clopidogrel, metoprolol, review of recent laboratory studies reveals that the
simvastatin, and lisinopril. Physical examination reveals patient has a moderate transaminitis and an elevated
diminished left-sided posterior tibialis and dorsalis fasting blood glucose level. An iron panel shows:
pedis pulses. Further, the left foot appears shiny and
hairless and, when elevated, displays an increase in pal- Blood, plasma, serum
lor. An ankle-brachial index reveals: Iron, serum 420 µg/dL
Ferritin, serum 1580 ng/mL
Left lower extremity 0.5
Iron-binding capacity Low
Right lower extremity 0.9
Based on these results, the most appropriate treatment
The man is not interested in surgery and mainly wishes to
for this patient is
be able to walk further than he currently can. The medi-
cal treatment most likely to improve his symptoms is (A) deferoxamine
(A) adding cilostazol (B) ferrous sulfate
(B) adding pentoxifylline (C) liver transplant
(C) increasing the dose of aspirin (D) penicillamine
(D) stopping clopidogrel (E) phlebotomy
(E) stopping metoprolol

29
USMLE Step 3 Assessment Exam

62. A 59-year-old man comes to the clinic for a routine 63. A 32-year-old man comes to the clinic for a routine visit.
health care checkup. He denies any recent change in his He has recently immigrated from China and wishes to
health, though his chronic bronchitis is, as always, both- be treated for his liver disease. In China, he was told that
ering him. He coughs throughout the winter, often pro- he has had hepatitis since birth, but has never been
ducing a thick, white sputum, and has done so for years. treated. Aside from occasional right upper quadrant
Despite this, he continues to smoke at least one pack of pain and fatigue, he reports otherwise being in good
cigarettes per day, though he has cut down. In addition health. He has a normal appetite and denies any fevers,
to chronic bronchitis, the patient suffers from hyperten- chills, nausea, vomiting, bleeding, weight gain or loss, or
sion controlled with a thiazide diuretic, dyslipidemia change in abdominal girth. Past medical history is sig-
well treated with a statin, and benign prostatic hyper- nificant for depression, currently treated with sertraline,
plasia. The man wishes to discuss cancer screening. He and, according to the patient, finally under reasonable
has had a colonoscopy, prostate examination, and, at his control. Vital signs and physical examination are unre-
request, has had a prostate specific antigen level checked, markable. Laboratory studies reveal a mild transamini-
all of which were normal. He is concerned about his tis with an AST of 58 U/L and an ALT of 62 U/L, an
lungs, however. The man realizes he is at risk for lung undetectable alfa-fetoprotein level, and a normal CBC
cancer and wants to know if he should “get some x-rays and set of chemistries. A hepatitis panel shows:
or something.” Physical examination and laboratory
studies are essentially unchanged from 6 months earlier. Hepatitis A antibody (IgG) Positive
The patient’s last chest radiograph was 4 years ago when Hepatitis B surface antibody Negative
he was diagnosed with bronchitis, and he has never had
Hepatitis B surface antigen Positive
a CT scan or a sputum cytologic examination. Based on
this patient’s presentation, the most appropriate lung Hepatitis B E antigen Positive
cancer screening strategy is Hepatitis C antibody Negative
(A) annual chest radiograph
An ultrasound of the liver reveals a mildly enlarged liver
(B) annual radiograph and sputum cytology with normal consistency and ductal architecture. Based
(C) annual sputum cytology on this patient’s presentation, the most appropriate ini-
(D) CT scan of the thorax every 5 years tial therapy is
(E) no additional screening examination necessary (A) interferon monotherapy
(B) interferon (pegylated) and ribavirin
(C) lamivudine (3TC) monotherapy
(D) no additional treatment necessary
(E) ribavirin monotherapy

30
Internal Medicine

64. A 52-year-old woman comes to the clinic complaining 65. A 60-year-old man with a history of ischemic car-
of flank pain. The pain comes in waves and radiates to diomyopathy comes to the emergency department
her groin. She describes it as unbearable and being an short of breath. Over the last week he has suffered sig-
11 on a pain scale of 1 to 10. The pain started earlier in nificant lower extremity edema and has had to sleep on
the day and has occurred intermittently, lasting for two pillows to minimize a feeling of suffocation. He
upwards of an hour. There has been no associated nau- admits to running out of his usual medications approx-
sea, vomiting, fever, or chills. She has never had pain imately a week earlier and has been too busy to get his
like this before, but does have a past medical history sig- prescriptions refilled. Examination reveals a man in
nificant for poorly controlled diabetes and recurrent moderate discomfort who has difficulty completing his
urinary tract infections. Vital signs are: temperature 38 sentences when speaking. Vital signs are: temperature
C (100.4 F), blood pressure 149/90 mm Hg, pulse 37 C (98.6 F), blood pressure 108/70 mm Hg, pulse
105/min, and respirations 20/min. The woman appears 105/min, and respirations 22/min. Auscultation of the
uncomfortable and seems unable to find a comfortable lungs reveals bilateral basilar crackles, reduced breath
spot on the examination table. There is no costoverte- sounds in both bases, and dullness to percussion over
bral angle tenderness, though she has mild suprapubic the lower thorax. An S3 gallop is heard and 2+ clubbing
tenderness. She states her right flank hurts, but the pain edema is present in both lower extremities. An electro-
changes only minimally with palpation. A dipstick uri- cardiogram is unchanged from a previous one 4
nalysis shows: months earlier, and a set of cardiac enzymes is negative.
A chest radiograph shows an enlarged cardiac silhou-
Urine ette, interstitial fluid extending to pleural surface,
pH 9.0 cephalization of the blood vessels, and mild to moder-
ate bilateral effusions. The intern working the emergen-
Glucose 1+
cy department performs a thoracentesis and sends the
Protein 3+ pleural fluid to the laboratory for analysis. The most
Nitrite 1+ likely finding is
Leukocyte esterase 3+ (A) hemorrhagic-appearing pleural fluid with many
Blood 3+ RBCs
(B) pale yellow, almost clear-appearing pleural fluid
An abdominal radiograph reveals what seems to be a
(C) pleural fluid with a low glucose and low pH
small stone just superior to the pelvic rim. Given this
patient’s presentation, the most likely cause of her stone is (D) a significantly elevated pleural fluid LDH level
(E) a total protein level similar to serum protein level
(A) Escherichia coli infection
(B) nephritic syndrome
(C) nephrotic syndrome
(D) proteus infection
(E) staphylococcal infection

31
USMLE Step 3 Assessment Exam

66. A 32-year-old woman is referred to your office because 67. A 62-year-old man comes to the clinic for a routine
of hypertension that is difficult to control with the health checkup. One month earlier, he was scheduled to
usual medications. When the patient is examined and have a cholesterol screening panel drawn. The laborato-
basic laboratory studies are performed, it is discovered ry switched the requisition form with another patient’s,
that she also has mild diabetes mellitus, irregular however, and instead tested the sample for thyroid
menses, easy bruisability, and osteoporosis. The family stimulating hormone (TSH) levels and free thyroxine
also remarks that she has developed wide mood swings, (T4) levels. The patient was notified of the mistake and
seeming to be euphoric at times, but having episodes of the cholesterol panel was redrawn. It was noted, however,
unexplained sadness at other times. Relevant negative that the TSH level was elevated at 20 mU/L, whereas the
findings include the fact that her serum potassium is T4 level was within normal limits, though nearing the
normal, she has no scalloping of her ribs on chest x-ray, low side of normal. The test was repeated today, con-
and there is no hint of any flank or upper abdominal firming the elevated TSH and normal T4 levels. The
bruits. A reasonable way to continue her workup at this patient, although he says he feels in his usual state of
point would be to do health, is concerned about these abnormal laboratory
values. A full review of systems fails to reveal any symp-
(A) aldosterone and renin measurements
toms that can be related specifically to thyroid disease,
(B) duplex scanning of her renal arteries though he reports vague symptoms of malaise and
(C) low-dose overnight dexamethasone suppression test arthralgias. His past medical history is significant for
(D) spiral CT scanning of the thoracic aorta coronary artery disease, with a myocardial infarction 5
years ago and a distant history of depression, but no
(E) urinary collection for catecholamines
autoimmune or thyroid disease. Vital signs and physical
examination are within normal limits. The most appro-
priate next step in the management of this patient’s
condition us to
(A) begin supplemental thyroxine (T4) treatment with
a goal of normal TSH levels
(B) check antithyroid peroxidase antibody levels, start
T4 treatment if elevated
(C) no further treatment is necessary; recheck TSH
and T4 levels in 1 year
(D) order a T3 level; if reduced, start treatment with
supplemental T3 immediately
(E) schedule followup appointment in 3 months,
recheck TSH levels at that time

32
Internal Medicine

68. A 48-year-old woman comes to the emergency depart- 70. A 29-year-old man comes to the emergency depart-
ment complaining of shortness of breath that was acute ment at 4 AM complaining of a terrible headache that
in onset, and also complains of lightheadedness. She began abruptly 2 hours earlier and awoke the patient
reports that she has a history of chronic obstructive from sleep. Acetaminophen and ibuprofen have barely
pulmonary disease and hypertension. Her medications affected the headache, and now his neck is beginning
include albuterol inhalers and hydrochlorothiazide. On to hurt. The headache, a 10/10, is located mainly on
examination, her pulse is 190/min and irregularly irreg- the right side of his head. The man is worried, as his
ular. Jugular venous distension is 18 cm of water. Blood father died from “some kind of stroke,” and he does not
pressure is 85/50 mm Hg. Chest examination reveals want the same to happen to him. Over the last week he
bilateral crackles. Cardiac examination reveals irregular has had a few severe headaches from which he does not
rhythm, systolic ejection murmur (3/6), and a loud left normally suffer, but none as severe as this. While you
ventricular S3. Chest radiograph is consistent with pul- are asking for more history, the man’s eyes suddenly
monary edema. Electrocardiogram reveals atrial fibril- roll backwards and he begins to convulse. Before you
lation with rapid ventricular response at 185/min. Prior can administer a milligram of lorazepam, he stops seiz-
electrocardiograms were in normal sinus rhythm. The ing and seems mildly confused as to where he is and
most appropriate management at this time is what all the fuss is about. A stat noncontrast CT scan is
obtained that reveals no evidence of bleeding. Given
(A) immediate direct current cardioversion
this patient’s presentation, the most appropriate next
(B) intravenous metoprolol test to diagnose this patient’s most likely condition is
(C) intravenous digoxin
(A) cerebral angiography
(D) intravenous diltiazem
(B) CT scan of brain with contrast
(E) intravenous adenosine
(C) lumbar puncture
(D) magnetic resonance imaging
69. A previously healthy 20-year-old college student comes (E) radiograph of head
to the clinic because of penile discharge, dysuria, and
urinary frequency. He had unprotected sexual inter-
course at a party 4 days before the onset of symptoms.
His temperature is 37.0 C (98.6 F), blood pressure is
110/60 mm Hg, pulse is 70/min, and respirations are
12/min. No penile lesions are noted. The remainder of
his physical examination is normal. Cultures from the
urethra are taken. Gram stain reveals gram-negative
diplococci in pairs. The most appropriate intervention
at this time is to
(A) treat empirically for gonorrhea
(B) treat empirically for gonorrhea and chlamydia
(C) treat empirically for gonorrhea and chlamydia and
give HIV prophylaxis
(D) treat empirically for gonorrhea, chlamydia, and
syphilis
(E) withhold treatment until cultures are obtained

33
USMLE Step 3 Assessment Exam

71. A 26-year-old man is involved in a serious motorcycle 72. A 57-year-old man comes to the clinic complaining of
accident and is thrown more than 50 feet, finally collid- constipation. For the past 6 months he has had signifi-
ing with a nearby parked car. Emergency medical ser- cant constipation, with an average of one bowel move-
vices bring him immediately to the hospital, where he is ment a week. His stools seem smaller in size, though he
found to be conscious but disoriented and in a great denies melena, mucus, or gross blood. A review of his
deal of pain. Skeletal survey reveals fractures of multi- diet reveals that he generally has a low-fiber, low-fluid
ple ribs, the vertebral body at T11, his right femur, and diet, at least part of which is due to a recent fad diet he
multiple pelvic fractures. He is stabilized and trans- has started. He reports a fairly sedentary lifestyle, hasn’t
ferred to the intensive care unit (ICU), where he devel- seen a physician in years, and does not take any pre-
ops hypoxemic respiratory failure and requires scription or over-the-counter medications. Review of
mechanical ventilation. On the second day of being in symptoms is unremarkable. Physical examination,
the ICU, his serum creatinine increases to 5.8 mg/dL including a rectal examination, is unrevealing. A com-
from its initial level of 0.9 mg/dL. Continuous plete blood count shows:
hemodialysis is begun and the patient remains stable.
One day later the nurse notes that the patient has begun Hematocrit 34%
to bleed from around intravenous lines and from trau-
Hemoglobin 11.1 g/dL
matic wounds that were previously sutured.
Nasogastric lavage yields frank blood. The study that is Leukocyte count 5500/mm3
most likely to be normal in this patient is
Mean corpuscle hemoglobin 21 pg/cell
(A) activated partial thromboplastin time
Mean corpuscle volume 74 µm3
(B) antithrombin III levels
Platelet count 483,000/mm3
(C) fibrinogen levels
(D) platelet count The most appropriate next step in management of this
(E) prothrombin time patient’s constipation is to
(A) order fecal occult blood testing
(B) perform a flexible sigmoidoscopy
(C) refer for a full colonoscopy
(D) start daily senna laxative treatment
(E) start a trial of fiber, fluids, and stool softener

34
Internal Medicine

73. A 70-year-old man with a history of myelodysplasia 75. A 65-year-old woman comes to the emergency depart-
and chronic anemia comes to the clinic complaining of ment with a chief complaint of acute onset of substernal
severe fatigue. He is pale, easily winded, and has a chest pain for the past 2 hours. She complains of increas-
hyperdynamic systolic heart murmur. Over the past 6 ing diaphoresis. She has a medical history that includes
months his myelodysplastic syndrome has become high blood pressure and diabetes. She currently is taking
worse, requiring him to have multiple transfusions to lisinopril, 10 mg daily, and glyburide, 5 mg daily. She has
maintain a safe hematocrit. Today he is found to have a no known medical allergies. Her blood pressure is 100/80
hematocrit of 21%. The patient is sent to the transfu- mm Hg and pulse is 60/min. Lungs sound clear. Heart is
sion center to receive a 2-unit transfusion of packed red regular. Electrocardiogram reveals 0.5 mm ST elevation
blood cells. However, once the patient is typed and in the inferior leads. She is admitted to the intensive care
screened, the blood bank calls to inform you that they unit for an acute inferior myocardial infarction. In the
do not have any appropriate blood available and will ICU, a pulmonary artery catheter is placed. Her wedge
have to search the Tri-County area for a suitable speci- pressure is 2 mm Hg, and right atrial pressure is
men. The patient has never had compatibility issues in 10 mm Hg. The most appropriate management is to
the past. The most likely reason for the difficulty in
(A) administer digoxin
finding appropriate blood for this patient is the
(B) administer dobutamine
(A) development of alloantibodies
(C) administer fluid boluses
(B) development of autoantibodies
(D) administer isosorbide mononitrate
(C) need for special filtering of blood
(E) place an intraaortic balloon pump
(D) presence of anti-HLA antibodies
(E) progression of myelodysplasia

74. A 42-year-old man comes to the clinic complaining of


severe abdominal pain that has developed over the past
week. The pain is alternating between dull and sharp
and is difficult to localize. Additionally, over the last
month the patient reports feeling weak, having inter-
mittent low-grade fevers, and having an unexplained
20-lb weight loss. When asked about his past medical
history, the patient states that he has been diagnosed
with “some type of hepatitis.” Review of systems reveals
significant arthralgias and myalgias but is otherwise
unremarkable. Physical examination reveals an uncom-
fortable, ill-appearing man with a blood pressure of
182/105 mm Hg. He has diffuse abdominal tenderness,
though no hepatosplenomegaly is appreciated.
Neurologic examination is remarkable for extremely
tender legs and a left foot drop and absent left lower
extremity reflexes. A mottled, flat purple rash is seen on
the patient’s left ankle. Routine laboratory studies are
remarkable for an elevated creatinine and a normocyt-
ic anemia. Given this patient’s presentation, the most
likely underlying condition is
(A) Hepatitis A
(B) Hepatitis B
(C) Hepatitis C
(D) Hepatitis D
(E) Hepatitis E

35
Internal Medicine Assessment Exam
Answers and Explanations

ANSWER KEY
1. D 26. A 51. D
2. E 27. C 52. D
3. A 28. D 53. E
4. B 29. E 54. D
5. B 30. B 55. B
6. A 31. A 56. A
7. B 32. B 57. A
8. C 33. B 58. A
9. C 34. E 59. D
10. E 35. B 60. A
11. D 36. C 61. E
12. D 37. D 62. E
13. E 38. B 63. C
14. A 39. E 64. D
15. C 40. D 65. B
16. E 41. B 66. C
17. D 42. B 67. A
18. D 43. B 68. A
19. C 44. D 69. B
20. C 45. A 70. C
21. D 46. B 71. C
22. D 47. E 72. C
23. D 48. E 73. A
24. B 49. D 74. B
25. B 50. C 75. C

37
USMLE Step 3 Assessment Exam

1. The correct answer is D. This patient has numerous intrahepatic portosystemic shunts (TIPS procedure) or
complications of diabetes and now seems to have devel- definitive treatment with liver transplantation.
oped diabetic nephropathy (and a possible type IV RTA, Famotidine (choice A) and omeprazole (choice D)
hinted at by the mild hyponatremia/hyperkalemia). increase the pH of the stomach, reducing the likelihood
Insulin is cleared renally. As patients develop renal fail- of gastric, not esophageal, bleeding. These agents,
ure, the insulin’s effective half-life increases, resulting in which work by way of H2-receptor blockade and pro-
a reduced need for exogenous insulin. Patients may find ton-pump inhibition respectively, can reduce the risk
their diabetes seemingly “cured” by their renal failure or, for recurrent bleeding from acute gastritis, but play no
if their insulin dose is not adjusted accordingly, may role in the management of esophageal varices.
have episodes of hypoglycemia.
Misoprostol (choice B) is a prostaglandin analogue. It
Insulinomas (choice A) are incredibly rare, on the order can be used to prevent NSAID associated gastritis in
of an annual four cases per 1 million people, and thus patients requiring NSAID treatment, but at significant
this is unlikely to explain this patient’s symptoms. risk for gastritis. It functions by replacing prostaglandin
Given that this patient has had long-standing diabetes normally produced by a COX-1 mediated pathway,
and now has proteinuria and an elevated creatinine which is blocked by NSAIDs. Prostaglandin is necessary
level, renal failure, resulting in a prolonged insulin to produce the mucous layer that protects the stomach
effect, seems a more likely scenario. lining from the acidic luminal environment.
There is no evidence that this patient has heart failure Octreotide (choice C) is a somatostatin analogue that
(choice B). The lack of any physical findings argues can be used in the acute management of esophageal
against heart failure, and the Q-waves on EKG simply bleeds. It also functions to reduce portal pressure,
reflect the presence of the patient’s previous myocardial mainly by inhibiting the release of splanchnic vasodila-
infarction. Heart failure, if associated with significant tors (such as glucagon). Splanchnic vasoconstriction
edema, can reduce or delay insulin absorption, and results in reduced portal blood flow and pressures.
usually results in hyperglycemia, not hypoglycemia. Because of its formulation (intravenous only), cost, and
Noncompliance (choice C) is common, but usually short half-life, its role is limited to emergency inpatient
results in hyperglycemia. Prolonged hyperglycemia treatment.
results in glycosylated hemoglobin (nonenzymatic
attachment of glucose moieties to red blood cell mem- 3. The correct answer is A. The first-line therapy for
branes). As such, one would expect an elevated osteoarthritis, particularly in elderly patients or in
hemoglobin A1c, not a level that has declined over time. those predisposed to bleeding, is acetaminophen.
Remember, hemoglobin A1c levels reflect glucose levels Acetaminophen does not increase the risk for a gas-
over the last 8 to 12 weeks. trointestinal bleed and has been found to be as effective
as NSAIDs in controlling the pain of osteoarthritis.
Resolution of underlying diabetes (choice E) is unlikely.
Further, it does not have the potential to interfere with
The natural history of diabetes is an increasing medica-
aspirin therapy (for coronary artery disease).
tion requirement. Patients first need oral hypoglycemics
and, as the disease progresses, require insulin. Given Capsaicin cream (choice B) is an adjunct that may pro-
the progression of this patient’s disease, it is unlikely that vide some relief in some patients. Given this patient’s
the condition has reversed itself spontaneously. multiple joint involvement, systemic analgesia is a more
appropriate first-line treatment. Additional noninva-
2. The correct answer is E. This patient has had an sive, nonsystemic treatments include exercise and phys-
episode of esophageal variceal bleeding, likely caused by ical therapy, hot and cold packs, and knee bracing.
underlying alcoholic cirrhosis. Cessation of alcohol Increasing this patient’s dose of aspirin (choice C) to
intake can result in some reversal of esophageal and analgesic doses, even though he takes an H2 blocker,
gastric varices. In addition, a nonselective beta-blocker will significantly increase his risk for bleeding. Likewise,
(commonly propranolol) can reduce portal pressure other NSAIDs, whether in analgesic (choice D) or anti-
and hence the risk for recurrent rupture of friable, dis- inflammatory doses (choice E) should be avoided if
tended esophageal veins. For patients who re-bleed possible, given this patient’s risk for gastrointestinal
despite prophylactic therapy, other treatment options bleeding.
include variceal sclerotherapy (which can be used as an
acute treatment or prophylactically), the addition of 4. The correct answer is B. This patient is having alcoholic
nitrites and alpha-blockers, or interventional and surgi- withdrawal symptoms and seizures and is progressing to
cal treatments. These treatments include transjugular delirium tremens. His presentation to the emergency

38
Internal Medicine Answers and Explanations

department and classic laboratory studies suggesting reversible anemia. Unfortunately, given this patient’s lim-
alcohol abuse (transaminitis, increased GGT, macrocytic ited exposure and failure to improve with cessation of the
anemia caused by hepatic dysfunction and/or malnutri- medication, he likely has a case of irreversible aplastic
tion, and hypokalemia and hypomagnesemia) should anemia. The treatment of choice for healthy young
not be missed; this patient needs withdrawal treatment patients is allogenic bone marrow transplantation.
with a benzodiazepine immediately. The patient should Immunosuppression with antithymocyte globulin and
have been started on a benzodiazepine on admission. cyclosporine (choice B) or other newer immunosup-
There is no indication for broad-spectrum antibiotics pressive regimens is indicated in patients who cannot
(choice A) or for returning to the OR (choice D). It is tolerate bone marrow transplantation, or as an adjunct
unlikely that this patient is suffering from an infection to transplantation. The primary treatment for healthy
or any other direct complication of his operation. young patients with irreversible aplastic anemia is bone
Beta-blockers and clonidine (choice C) are adjunct marrow transplantation.
treatments for alcohol withdrawal that may be used in GCSF (choice C) and erythropoietin (choice D) are
addition to benzodiazepines to treat adrenergic hyper- unlikely to provide much benefit. These agents stimu-
activity. The first-line treatment, however, is a benzodi- late marrow precursors, which are absent in aplastic
azepine such as lorazepam. anemia. The treatment of choice is replacement of dys-
Phenytoin (choice E) is of limited effectiveness in treating functional marrow.
withdrawal seizures and should not be used as prophylax- Watchful waiting and transfusions (choice E) are not
is in these patients. If the patient progresses to status appropriate treatments for this irreversible condition.
epilepticus, intravenous phenytoin or fosphenytoin may Transfusions may temporarily replace some blood
be used in addition to a benzodiazepine. components, though the risks from transfusions (infec-
tion, reaction) and inevitable sensitization limit this
5. The correct answer is B. The indications for throm- approach. Further, the patient is also leukopenic and at
bolytics include chest pain starting within the last hour risk for infection. The best treatment is bone marrow
and either an ST segment elevation, not depression transplantation as soon as possible.
(choice E), of >1 mm, or a new-onset left bundle
branch block (LBBB). An LBBB pattern makes inter- 7. The correct answer is B. Among the different variants
preting ST segments, indications of cell injury and of acute myeloid leukemia (AML), the one to know is
death, extremely difficult and is often, in and of itself, acute promyelocytic leukemia, classified as M3. This
an indication of coronary ischemia. particular variant, which accounts for 15% of AML, is
Diffuse T-wave inversions (choice A) are not an indica- notable because of its unique clinical manifestations
tion for thrombolytics. T-wave inversions can indicate and different treatment strategies from the other forms
ischemia, but may also be normal variants or may be of the condition. It is strongly associated with DIC
caused by noncardiac events. Metabolic conditions, induced by a cancer procoagulant and may present as
such as hypokalemia, can result in T-wave flattening, pulmonary or cerebral hemorrhage in up to 40% of
and intracranial bleeds may produce flipped T waves patients. Additionally, this variant responds well to all-
(cerebral T waves). trans-retinoic acid, which induces differentiation and
maturation of the promyelocytes. Patients can achieve
Q-waves (choice C) indicate old infarction. As such, Q
long-term remission with treatment. The other forms
waves are not an indication for thrombolytics. Stunned
of AML, including M0 (choice A), M5 (choice C), M6
ischemic myocardium requires urgent reperfusion.
(choice D), and the extremely rare M7 (choice E) do
There is no compelling evidence to justify thrombolytic
not have these defining characteristics.
therapy beyond 12 hours.
Unlike an LBBB, a right bundle branch block indicated 8. The correct answer is C. Heart block in a patient with
by the R-R′ morphology (choice D) is not an indication infectious endocarditis is an extremely worrisome sign, as
for thrombolytics. Although a right bundle is concern- it often indicates extension into the area surrounding the
ing, it does not render the ST segments uninterpretable heart valve. A ring abscess, not easily seen on transtho-
and does not carry as serious a prognosis as an LBBB. racic echocardiography, may present only as mild con-
duction delays. Indications for surgical treatment of
6. The correct answer is A. This patient has aplastic ane- infectious endocarditis include heart failure, persistent
mia, almost certainly caused by the use of chloram- infection or emboli on antibiotics, most prosthetic valve
phenicol. Chloramphenicol can produce an idiosyncrat- endocarditis, fungal endocarditis, and the presence of
ic and irreversible aplastic anemia and a dose-dependent abscess or involvement of the conduction system.

39
USMLE Step 3 Assessment Exam

Cultures indicate a highly resistant organism (choice 10. The correct answer is E. This patient has aspirin-sensi-
A), MRSA. Most MRSA can be treated with van- tive asthma, which accounts for up to 10% of cases of
comycin, daptomycin, linezolid, and quinupristin/dal- asthma. Clues include the age of onset, usually the third
fopristin and occasionally other antibiotics, however, or fourth decade, the presence of nasal polyps, and the
depending on the sensitivities of the organism. Only if predominance of rhinosinusitis symptoms related to
the patient remains febrile on treatment is resistance an NSAID use. Inhibition of COX-1 results in a relative
indication for surgical treatment. imbalance of leukotrienes and prostaglandins. The rel-
Surgical treatment may be indicated at this time ative excess of leukotrienes results in severe bron-
(choice B). Further imaging with a transesophageal chospasm. Given the pathophysiology of this condition,
echocardiogram may help delineate the extent of this leukotriene modulators, such as Zafirlukast or mon-
patient’s vegetation. telukast, should form the cornerstone of therapy, in
addition to a beta-agonist inhaler for quick relief. This
Embolic events (choice D), if occurring while on
is in contrast to other forms of asthma, where these
appropriate antibiotics, are usually considered an indi-
agents are generally used as steroid-sparing agents in
cation for surgical treatment of endocarditis. This
cases of severe disease. Of note, the obvious therapy is
patient has septic emboli to the lungs, but these
to avoid exposure. However, many patients may need
occurred before the initiation of antibiotics.
NSAIDs (e.g., osteoarthritis, cardioprotective baby
Right-sided lesions and regurgitation (choice E) are not aspirin, pain) or are at risk for inadvertent exposure
an indication for surgery. If there is direct valve erosion from medications that contain an NSAID. These
rather than a mechanical regurgitation, surgical treat- patients need leukotriene inhibition as well as desensi-
ment is usually necessary. tization with daily aspirin use.
9. The correct answer is C. This patient has an acute inter- Aminophylline and theophylline (choice A) are rarely
stitial nephritis (AIN), likely caused by ampicillin expo- used. They cause modest bronchodilatation and may
sure. Given his pre-existing renal insufficiency, he has strengthen diaphragmatic contractions. However, they
now become hypervolemic and has a uremic pericarditis, have a narrow therapeutic window, have a wide variety
an indication for emergent hemodialysis. Most patients of drug interactions, and are generally less effective
recover from AIN once the offending agent is removed, than other agents available to treat the condition. These
though some have continued symptoms and require agents have no particular benefit in aspirin sensitive
steroids or other immunosuppressives. In the interim, asthma.
however, this patient needs dialysis to reverse his peri- Epinephrine (choice B) can treat any form of asthma,
carditis and deal with his volume status. but is not a first-line treatment, as its hemodynamic
A fluid bolus (choice A) is risky in this patient, given his effects can be severe. Epi-pens are generally provided to
elevated jugular venous pressure and reduced renal patients at risk for anaphylaxis induced by bee stings.
function. The pericardial friction rub is an indication Nedocromil (choice C) is a mast cell stabilizer. It is used
that dialysis is necessary. prophylactically in exercise-induced and allergic asth-
Broad-spectrum antibiotics (choice B) are not indicated. ma. The inhaler is most useful in the pediatric popula-
This patient is suffering from a drug reaction, specifically tions, and less effective in adult populations.
AIN. Leukotriene inhibitors provide greater benefit for
patients with aspirin sensitive asthma.
Renal biopsy (choice D) is the gold standard to diag-
nose AIN, but it is not necessary in patients with a clas- Salmeterol (choice D) is a long acting beta-2 agonist. Its
sic presentation. Rather, the offending agent should be primary benefit is in the treatment of nocturnal symp-
removed and, if necessary, steroids started. Patients toms and cough-variant asthma. There is no need to
with severe renal failure may need temporary dialysis as provide a long-acting beta-2 agonist to someone with
their kidneys recover. intermittent symptoms.
Furosemide (choice E) is not indicated. There is no dif- 11. The correct answer is D. This patient meets the criteria
ference in outcomes (in terms of renal function) for Alzheimer dementia. Diagnosis does not require
between oliguric and nonoliguric renal failure. The invasive testing or advanced neurodiagnostic studies
most appropriate immediate treatment for this (choice E), but can be made in the office based mainly
patient’s condition is dialysis. on history and a mini-mental status examination. The
diagnostic criteria set forth in the DSM-IV include the
following four points:

40
Internal Medicine Answers and Explanations

1) the gradual and progressive decline in cognitive Broad-spectrum antibiotics, such as ceftriaxone (choice
function that results in impairment in social or occu- B), are not indicated in this patient, who has no obvi-
pational function; ous foci of infection. This patient’s low-grade fevers are
2) impairment in recent memory, disturbance of lan- entirely consistent with acute hepatitis. Rupture of hep-
guage, inability to execute skilled motor activities in atic Kupffer cells (macrophages), which are rich in
the absence of weakness, disturbance of visual pro- interleukin-1, a cytokine responsible for febrile reac-
cessing, or disturbance in executive function (includ- tions, presumably plays a role in the pathogenesis of
ing abstract reasoning and concentration); hepatitis-related fever.
3) the deficits must not occur exclusively in the setting Octreotide (choice C) is a somatostatin analogue that
of delirium; can be used in the acute management of esophageal
bleeds. It lowers portal pressure mainly by inhibiting
4) the cognitive deficits are not caused by other illnesses.
the release of splanchnic vasodilators such as glucagons,
If there is any doubt about a patient meeting these cri- thus reducing portal blood flow. There is no role for the
teria, further testing to rule out other conditions is war- use of octreotide in this patient.
ranted, as the diagnosis can be devastating. Patients Propranolol (choice E) is used as prophylaxis against
often receive vitamin B12, thyroid function studies, esophageal bleeds in patients with known esophageal
VDRL (or other neurosyphilis screen), and occasion- varices. It plays no role in the management of acute
ally, neuroimaging studies, though these studies rarely hepatitis. This patient does not have evidence of chron-
reveal a reversible cause of dementia. ic liver disease and cirrhosis, though his current behav-
Brain biopsy (choice A) can confirm Alzheimer demen- ior is certainly likely to result in such conditions if
tia, but is not necessary to make the diagnosis. Given unchanged.
the expense and morbidity of a brain biopsy, particu-
larly in a patient already facing a neurocognitive deficit, 13. The correct answer is E. This patient likely has Reiter
the test is not warranted. syndrome or a reactive arthritis. Reiter syndrome is
defined by the triad of uveitis, urethritis, and arthritis
The pathogenesis of Alzheimer disease in part includes a
and is most common in younger men. Additional
relative deficiency of acetylcholine. Testing for CNS
lesions classic for reactive arthritis, of which Reiter syn-
acetylcholine levels (choice B), however, is not routinely
drome can be considered a subset, include keratoderma
performed.
blennorrhagica (which this patient has), circinate bal-
Magnetic resonance imaging (choice C) can rule out anitis, and oral/genital ulcers. Enthesitis, sausage digits,
other conditions, particularly the rare tumor. CT scan and inflammatory back pain also may be present. These
also can be used and is generally a cheaper and quicker conditions are associated with nongonococcal urethritis
way to evaluate for a subdural or mass lesion if one is (Chlamydia and Ureaplasma) and infectious diarrhea
suspected. It is not necessary to image the brain to diag- (Campylobacter, Shigella, and Salmonella).
nose Alzheimer dementia.
This patient’s condition, a reactive arthritis, is not asso-
12. The correct answer is D. This patient likely has alcoholic ciated with a family history of malignancy (choice A) or
hepatitis and alcoholic fatty liver, which often coexist. As a family history of skin disease (choice B).
the liver fails to produce clotting factors, patients devel- Occupational history and exposures (choice C) should
op a severe coagulopathy with a markedly elevated always be evaluated when trying to determine the cause
PT/INR. Among patients with severe coagulopathy and of a rash with discrete margins, as a contact dermatitis
elevated bilirubin, steroids have been found to improve often is to blame. This patient’s particular constellation
survival. If the patient survives, much of the damage of symptoms is classic for a reactive arthritis, however,
from acute alcoholic hepatitis and alcoholic fatty liver is often seen after a sexually transmitted disease or an
reversible. Of note, prednisolone is the preferred agent, episode of infectious diarrhea.
as prednisone requires hepatic conversion (presumably Previous dermatologic complaints (choice D) are
limited in severe hepatitis) to prednisolone. important if one suspects psoriatic arthritis. Psoriatic
Intravenous albumin (choice A) is sometimes used in the arthritis is often associated with marked psoriasis
management of cirrhotic patients, but is not indicated in (patchy, silvery scaled lesions). Pitting of the finger-
the treatment of acute hepatitis. Its role is controversial, nails is a fairly sensitive indicator of patients who will
though generally it is believed to improve outcomes if progress to the arthritic form of the disease, which
used in cirrhotic patients with spontaneous bacterial often affects distal joints, in particular the distal
peritonitis. interphalangeal (DIP) joint. This patient’s dystrophic

41
USMLE Step 3 Assessment Exam

toenails are caused by the keratoderma blennorrhagi- makes using other agents, whether amiodarone, dilti-
ca, and his fingernails are normal. azem, or metoprolol (choice B) difficult, as all of these
medications cause hypotension.
14. The correct answer is A. This patient has the most
A transvenous pacemaker (choice D) is not warranted
common coronary heart disease (CHD) equivalent,
for atrial tachyarrhythmias. Rather, episodes of com-
diabetes mellitus. ATP III guidelines recommend that
plete heart block or prolonged symptomatic pauses jus-
these patients have a goal low-density lipoprotein
tify pacemaker placement.
(LDL) of less than 100 mg/dL. If this patient’s LDL level
were near goal, diet and exercise might be adequate to If time permits, a transesophageal echocardiogram,
reach this target. This patient deserves therapy with an precardioversion anticoagulation, and then electrical
HMG-CoA reductase inhibitor (statin), however, for cardioversion (choice E) is a reasonable strategy. This
two reasons. First, she is not near goal. Patients with patient’s blood pressure is not adequate to perfuse his
CHD or CHD equivalents with an LDL of 130 mg/dL or organs, however, and he is having chest pain.
greater should start lipid-lowering therapy. Second, Cardioversion should not be delayed for other inter-
there is mounting evidence (specifically, the Heart ventions or medications.
Protection Study) that patients with diabetes benefit
16. The correct answer is E. This patient has a classic pre-
from statin therapy regardless of LDL level.
sentation of aspiration pneumonia. Aspiration pneu-
Exercise and dietary changes (choice B) are unlikely to monia, in contrast to a chemical aspiration pneumonitis
be successful in this patient. First, she is already on a (from acidic gastric fluid), is usually caused by indolent
diabetic diet and exercise plan that presumably should anaerobic oral flora that usually resides in the gingival
provide some benefit. Second, given her level of LDL crevices. Radiographs show infiltrates predominantly in
cholesterol (≥130 mg/dL), she warrants lipid-lowering the right mid or lower lung zones and often display areas
drug therapy. of necrosis. The preferred agent is clindamycin, which
There is no need to increase metformin (choice C). Her provides good anaerobic and gram-positive coverage,
hemoglobin A1c, which is a better predictor of glycemic though combination treatment with metronidazole and
control than a one-time fasting glucose, is within penicillin is also a proven regimen.
acceptable limits for a patient with diabetes (common- Trimethoprim/sulfamethoxazole (TMP/SMX) and
ly less than 7%). Further, metformin and improved prednisone (choice A) is a combination commonly
glycemic control will only indirectly affect her lipid used for PCP pneumonia. It is not appropriate for
profile. Her lipid panel warrants starting medical treat- anaerobic infections, as TMP/SMX has little to no effect
ment now. against anaerobes, and resistance rates among gram-
Goal LDL for this patient is less than 100 mg/dL. As positive organisms are high.
such, further action (choice D) is warranted now, not in A high-resolution CT scan (choice B) or further evalua-
1 year. tion with bronchoscopy (choice C) is not necessary. This
Gemfibrozil (choice E) is not the best choice for this patient has a classic presentation for anaerobic aspira-
patient. If she had low HDL or high triglycerides, then a tion pneumonia. She has a predisposing factor—a
fibrate would be an ideal first choice. Given the potency recent CVA. Her coughing at dinner probably represents
of the statins in lowering LDL cholesterol and possible continued aspiration. Further, the purulent sputum is a
non-cholesterol-mediated benefits in CHD and diabetic big clue to the presence of anaerobes.
patients, simvastatin is a better first choice agent. Ceftriaxone and azithromycin (choice D) is an appropri-
ate combination for community-acquired pneumonia. It
15. The correct answer is C. This patient is not in a stable
is not proven for aspiration pneumonia; clindamycin is
rhythm. Whether multifocal atrial tachycardia (MAT,
the standard of care.
more common in patients with pulmonary disease) or
atrial fibrillation, the treatment algorithms are similar. 17. The correct answer is D. This patient has a classic pre-
This patient’s low blood pressure, anginal pain, and sentation of idiopathic thrombocytopenic purpura
orthostatic symptoms warrant emergent cardioversion. (ITP), an immune-mediated destruction of platelets.
Once this patient is cardioverted, amiodarone (choice ITP is seen in pediatric populations and in young
A) may be appropriate to maintain sinus rhythm. This women and may be initiated by a viral infection or may
patient’s condition warrants electrical cardioversion, be associated with a variety of autoimmune conditions.
however, which is more successful than chemical car- The typical presentation, at least for the purposes of
dioversion. Further, the patient’s low blood pressure taking boards, is an otherwise healthy patient with

42
Internal Medicine Answers and Explanations

petechiae and bleeding, usually oozing, not gushing, Followup should be scheduled at routine times, such as
and an isolated decrease in platelets (i.e., other cell every 3 months, not as needed or in the distant future
counts are near normal). The first line treatment is a (choice C). It is necessary to check repeated blood
steroid for 4 to 6 weeks, followed by gradual taper. smears and perform serial clinical examinations to
DDAVP or factor VIII replacement (choice A) is useful monitor for signs of disease progression.
for treating von Willebrand disease. These patients pre- Prednisone (choice E) is an additional regimen that can
sent with “platelet-type bleeding,” specifically petechiae, be used alone or as part of a CHOP (cyclophos-
gingival bleeding, and difficult to control epistaxis, for phamide, doxorubicin, vincristine, and prednisone)
example, but they have normal platelet counts. Bleeding regimen. It is particularly effective for patients with pre-
time, which effectively measures the time of clot forma- dominant autoimmune manifestations, such as
tion following a skin puncture, will be elevated. hemolytic anemia, or autoimmune-mediated thrombo-
Platelet transfusions (choice B) should be avoided if the cytopenia. Fludarabine is an additional agent also used
patient is not bleeding uncontrollably. Transfusions can to treat chronic lymphocytic leukemia.
increase the production of autoantibodies and, if the
19. The correct answer is C. This patient has a cellulitis by
patient is not bleeding, are not necessary. Most patients
history and clinical examination. Nothing further is
can tolerate platelet counts as low as 10,000/mm3 with-
necessary to make the diagnosis. Predisposing factors in
out spontaneous bleeding, though there are no clear
this patient include venous insufficiency, a skin break
transfusion cutoffs.
(tinea), and some degree of immunocompromise and
IVIG (choice C) is extremely expensive and offers an reduced circulation from diabetes. In diabetic patients,
additive advantage to prednisone. It is generally a empiric coverage for gram-positive and gram-negative
second-line treatment, though one that works quickly, organisms is necessary. Oral second generation
used if an acute increase in platelet counts or if a large cephalosporins or an oral penicillin with a beta-
platelet transfusion is required. Its ability to bind lactamase inhibitor (amoxicillin/clavulanate) often are
autoantibody and to inhibit splenic filtering will pro- used.
long the functional half-life of circulating platelets.
A full laboratory workup, such as blood count,
Fresh frozen plasma (choice E), also known as FFP, is chemistries, and blood cultures (choice A) is not indi-
not necessary, as this patient has a normal PT and PTT. cated. The patient can be treated empirically for celluli-
FFP is used to correct clotting factor deficiencies, not tis. If the patient appeared systemically ill, then this
thrombocytopenia. workup, in addition to a surgical biopsy to rule out
necrotizing fasciitis, may be appropriate.
18. The correct answer is D. This patient has chronic lym-
phocytic leukemia, a disease predominantly affecting Ultrasonography (choice B) and venography (choice E)
people over age 50 years. The condition can present can be used to evaluate for deep venous thrombosis,
with dramatic symptoms, such as recurrent infections, which presents in a similar fashion. This patient is anti-
hemolytic anemia, bulky, painful lymphadenopathy coagulated and does not have any obvious clinical clues
and splenomegaly, or classic B symptoms (fevers, night to suggest a DVT. As such, neither study is warranted at
sweats, and weight loss). Up to one quarter of patients, this time. In general, an ultrasound, which avoids
however, have an indolent course with few to no symp- nephrotoxic contrast dye, is the preferred of the two
toms. In these patients, watchful waiting is the most studies for evaluating deep venous thrombosis.
appropriate therapy, particularly in older patients who Skin biopsies (choice D) have a low yield when used to
are less likely to tolerate treatment and have less (in diagnose cellulitis and are not recommended.
terms of life span) to gain from treatment. In this
patient, a repeat CBC should be performed to assess 20. The correct answer is C. This patient has rhabdomyoly-
doubling time of the WBC count. If the patient has sis, or muscle breakdown, in this case caused by pro-
worsening symptoms or a rapid doubling time, treat- longed prone positioning. The fundamental pathophysi-
ment should be started. For the time being, however, ology involves the release of intracellular contents and
watchful waiting is appropriate. pigment dye, which results in massive electrolyte abnor-
malities and pigment-induced renal tubular damage.
A common single-agent treatment for chronic lympho-
Calcium salts (such as with phosphate) form and deposit
cytic leukemia is chlorambucil, though there is no evi-
in damaged tissue, resulting in significant hypocalcemia
dence that favors either high-dose (choice A) or low-dose
that needs to be monitored and corrected in patients with
(choice B) regimens.
rhabdomyolysis.

43
USMLE Step 3 Assessment Exam

Release of intracellular contents results in a metabolic 22. The correct answer is D. This patient has Guillain-
acidosis. The pH is expected to decrease, and one Barré syndrome, characterized by the three As of paral-
should not find a high arterial blood pH (choice A) if ysis: acute, areflexic, and ascending. The condition
there is significant rhabdomyolysis. often follows a recent Campylobacter or herpes virus
Rhabdomyolysis results in renal tubular obstruction infection, though it may occur without obvious precip-
from heme pigment casts and tubular injury from free itating illness. The mortality rate is 3-4% and is caused
chelatable iron. The myoglobin in the urine will cross- by respiratory failure from paralysis of respiratory mus-
react with urine dipstick assays for hemoglobin. cles. As such, respiratory support and early intubation
Microscopic analysis, however, will reveal a low, not are advisable. Additional treatments shown to improve
high, urine red blood cell count (choice B). outcomes include plasmapheresis and immunoglobulin
treatment (IVIG).
Phosphate (choice D) and potassium (choice E) are
usually stored intracellularly. Severe rhabdomyolysis Antibiotics (choice A) and antitoxin (choice B) are use-
results in release of these cellular constituents from ful in the treatment of botulism, which also can present
damaged cells. These electrolyte abnormalities can be with an acute paralysis that may result in respiratory
severe and need to be monitored. Severe rhabdomyo- failure. The key difference is that botulism is a descend-
lysis may require emergent dialysis. ing paralysis; the cranial nerves are usually the first part
of the nervous system affected. A history of intravenous
21. The correct answer is D. Asthma is a symptomatically drug use (which predisposes to wound botulism) or
intermittent condition, and thus has intermittent PFT home-canned food is a big clue to the diagnosis.
abnormalities. This patient, with exercise-induced asth- Interferon (choice C) is useful for the treatment of mul-
ma that is worsened by cold weather, is currently without tiple sclerosis. Lesions often occur in the descending
airflow limitation, as evidenced by a normal examination tracts or the brain and result in upper motor neuron
and lack of acute symptoms. The PFTs will be normal signs, mainly spasticity and focal neurologic deficits,
but, given the patient’s sensitive airways, will display evi- rather than a diffuse and ascending paralysis. This
dence of bronchospasm after the administration of patient is, however, at approximately the age when mul-
methacholine, as cholinergic stimulation results in air- tiple sclerosis often first presents.
way constriction.
Steroids (choice E) have not been shown to be effective
This patient should have normal flow rates using a peak in the treatment of Guillain-Barré syndrome.
expiratory flow meter (choice A), not elevated or
depressed rates. He is currently asymptomatic, and thus 23. The correct answer is D. This patient has carpal tunnel
does not likely have significant airflow obstruction. A syndrome. Although the condition is often idiopathic, it
peak flow meter is a simple device patients can use to also is associated with endocrinopathies, such as
monitor the severity of their asthma symptoms. As hypothyroidism and acromegaly. The bilateral presenta-
bronchospasm worsens, peak flows decline. tion should raise suspicion for an underlying systemic
In an acute attack, one would expect an FEV1/FVC ratio cause of her carpal tunnel syndrome. In addition to
of less than 60 to 70% of the predicted value (choice B). endocrine disorders, the condition also is associated with
Air is trapped behind constricted bronchi and takes a amyloidosis, pregnancy, and rheumatoid arthritis. The
long time to be fully exhaled. As such, the FEV1 declines patient’s other complaints, such as fatigue, constipation
to a greater amount than the FVC. The patients can still (perhaps misdiagnosed as irritable bowel syndrome),
get air out (the FVC); it just takes longer (the FEV1). and weight gain, may be caused by hypothyroidism.
A reduction in the total lung capacity, with preserved Antinuclear antibody titers (choice A) are not indicated.
FEV1/FVC ratio (choice E) indicates a restrictive, not There is no reason to suspect that this woman has an
obstructive, defect. A restrictive defect, such as obesity autoimmune disease.
or lung fibrosis, limits the expansion of the lung, thus A hemoglobin A1c level (choice B) is not indicated.
reducing lung volume, but does not limit the rate of Diabetes can present with neuropathic pain in a stocking-
exhalation. As such, the FEV1 and FVC are proportion- glove manifestation, though this patient has a description
ally reduced. more classic for carpal tunnel syndrome. If diabetes is sus-
An increase in the DLCO (choice C) is seen in an acute pected, the appropriate test for diagnostic purposes is a
attack, as high negative thoracic pressures bring more fasting glucose level or an oral glucose tolerance test.
blood into the lungs, resulting in greater capillary Additional testing (choice C) is necessary. Given this
recruitment and blood flow. In this patient, the DLCO patient’s bilateral presentation and particular complaints,
will be normal. diagnosis and treatment of hypothyroidism will help

44
Internal Medicine Answers and Explanations

resolve her symptoms, likely more effectively than splint- Diarrhea (choice A) is a result of hyperthyroidism,
ing, steroid injections, physical therapy, and ergonomic regardless of the underlying cause of the condition. It is
redesign, all commonly prescribed noninvasive treat- not a specific finding in Graves disease.
ments for carpal tunnel syndrome. Goiter (choice C) is not specific to Graves disease,
If this woman were younger, a urine pregnancy test occurring in a variety of causes of hyperthyroidism.
(choice E) may be reasonable. She is unlikely to be Further, a goiter may occur in euthyroid or hypothyroid
pregnant, however, and has symptoms that warrant patients.
thyroid functional studies. Hyperreflexia (choice D) is a result of hyperthyroidism,
regardless of the underlying cause of the condition. It is
24. The correct answer is B. This patient has classic symp-
not a specific finding in Graves disease.
toms of gastroesophageal reflux disease (GERD).
Patients with a typical presentation can be tried on Lid lag and stare (choice E) are distinct from exoph-
empiric acid suppression, such as omeprazole (now thalmos. Exophthalmos is a protrusion of the eyes
available in over-the-counter formulations). If there are resulting from a complex host response to circulating
any warning signs, such as presentation at an advanced TSH receptor antibodies. TSH receptor antibodies
age, weight loss, abdominal pain, bleeding, or systemic induce retro-orbital connective and adipose tissue
symptoms, then an upper endoscopy (choice D) should accumulation and the accumulation of glycosamino-
be performed before an empiric trial is started so that glycans. This excess tissue displaces the globe forward.
more serious conditions that may mimic or cause This is in contrast to a stare and lid-lag, which are
GERD, such as gastric and esophageal carcinoma, can caused by lid retraction from sympathetic hyperactivi-
be ruled out. ty seen in all forms of hyperthyroidism.
Testing for Helicobacter pylori, whether with a breath 26. The correct answer is A. Unfortunately, this patient has
urease test (choice A) or blood or stool tests, is not nec- required a massive blood transfusion for a likely diver-
essary for the management of GERD. The role of H. ticular bleed. Although these bleeds often stop on their
pylori in GERD is unclear; some evidence suggests that own, they can continue, requiring surgical intervention
H. pylori seropositivity may correlate with lower rates of (usually preceded by colonoscopy or radionucleotide
GERD, though the mechanism of such an association is scintigraphy to localize the site of bleeding) or angiog-
not clear. In any case, testing and treating H. pylori with raphy with embolization. Packed red blood cells con-
an appropriate regimen such as omeprazole, amoxi- tain citrate to prevent coagulation during storage.
cillin, and clarithromycin (choice E) is not part of the Citrate can bind calcium, resulting in low levels of ion-
management of GERD. ized free calcium (though total levels may be normal).
Lifestyle modifications, such as cessation of smoking, This can present as a prolonged QT interval, tetany, or
limiting alcohol intake, eating small meals for dinner, frank seizures. Ionized calcium levels or an EKG should
and elevating the head of the bed, can help modestly be checked as the transfusion total approaches 10 U.
reduce GERD symptoms. Avoiding spicy food (choice Massive transfusions can result in dilution of coagula-
C), however, has not been shown to be helpful. After all, tion factors, thus requiring the additional transfusion
the acid causing the pain is derived from gastric parietal of fresh frozen plasma (choice B). The cause of this
cells, not from the consumed food. patient’s seizure is low ionized calcium, however;
replacement of calcium is necessary now.
25. The correct answer is choice B. This patient has Graves
disease, the most common cause of hyperthyroidism in Typical treatments for seizures, including lorazepam
the United States. Among the causes of hyperthy- (choice C) and phenytoin (choice D), are not necessary.
roidism, findings unique to Graves disease include If this patient’s calcium is replaced, he will not continue
exophthalmos, periorbital and conjunctival edema, to suffer seizures.
limitation of ocular motion, and infiltrative dermopa- Massive blood transfusions may be associated with
thy (pretibial myxedema). These findings are caused hyperkalemia; giving empiric potassium (choice E) is
directly by host responses to the presence of TSH recep- not recommended. Stored blood leaks potassium and
tor antibodies rather than the resulting hyperthy- the older the unit transfused, the higher the potassium
roidism. Up to 70% of patients with Graves disease have level. This is not usually a concern, except when large-
exophthalmos, making it an extremely useful clinical volume transfusions are required.
finding.

45
USMLE Step 3 Assessment Exam

27. The correct answer is C. Polymorphic ventricular 29. The correct answer is E. A noncontrast spiral CT scan
tachycardia, also known as Torsade de Pointe, is associ- is one of the most sensitive tests to detect nephro-
ated with hypomagnesemia. Empiric magnesium lithiasis. The test has the advantage not only of high
replacement in these patients, in addition to electrical speed, availability, and excellent sensitivity, but also
defibrillation, can be effective in converting these avoids the contrast dye necessary for older tests, such as
patients to a more stable rhythm. an IVP. As a stone is radiopaque, it appears best in the
Junctional tachycardia (choice A) and pulseless electri- ureter without contrast. Radiopaque dye can bypass the
cal activity (choice D) are not treated with magnesium. obstruction and light up the entire ureter, or at least
Magnesium is used primarily for primary ventricular obscure the margins of the stone, making the film more
arrhythmias, and is particularly efficacious when treat- difficult to interpret.
ing Torsade de Pointe. According to the ACLS algorithm, Abdominal and renal ultrasound (choice A) is a good test
magnesium may be tried in monomorphic ventricular to search for renal stones. A CT scan is more sensitive,
tachycardia (choice B), though epinephrine, vaso- however, particularly in obese patients. In obese patients,
pressin, and lidocaine should be considered first. ultrasound is usually technically limited by the large
Ventricular fibrillation (choice E) is unlikely to respond amount of fat that the sound waves must penetrate.
to magnesium. If any response is to be had, it is most Additionally, a CT scan can see a stone in the pelvis,
likely with a combination of electrical defibrillation and which can be difficult to visualize with an ultrasound.
either epinephrine or vasopressin. Contrast enhanced CT scans (choice B) are not indicated
when searching for kidney stones.
28. The correct answer is D. Hydroxyurea increases the
An IVP (choice C) should be avoided in this patient
proportion of hemoglobin F. Increased levels of
with baseline compromised renal function. The large
hemoglobin F are partially protective against sickling
dye load can result in worsening renal function and, in
and decrease the frequency of painful vaso-occlusive
an already predisposed patient, may result in the need
crises. Hydroxyurea is used in the long-term treatment
for dialysis.
of sickle cell disease and is not used in the acute setting.
A KUB (choice D) has a sensitivity of up to 80% when
Exchange transfusion (choice A) is used in the acute
interpreted by a skilled radiologist. Ultrasound (in the
setting and not in the long-term management of sickle
right patients) and CT scan, however, have significantly
cell disease. For patients who fail to respond to oxygen,
greater sensitivity.
hydration, and pain control and who continue to have
significant hypoxia, consider exchange transfusion 30. The correct answer is B. Patients with one pathologic
rather than simple blood transfusions (always a contro- fracture from multiple myeloma are at risk for repeat
versial topic in the management of sickle cell patients). fractures. A complete skeletal survey is necessary to look
Folate supplementation (choice B) is important in for lytic lesions, caused by plasma cells actively secreting
managing the anemia associated with sickle cell disease. IL-6, an osteoclast activating cytokine. Lesions can be
Chronic hemolysis and cell turnover can quickly treated surgically (rods and pins) or with external beam
exhaust the body’s stores of folate, and thus can com- radiation. The best study is a full body skeletal survey
pound an already severe anemia. Folate will not, how- with conventional radiographs.
ever, reduce the frequency of sickle cell crises. CT scan (choice A) provides a great deal of information
Hydroxyurea is the primary agent for this purpose. about internal structures, but is technically not as easy
Although hypoxia can induce a sickle cell crisis, there is to review when searching for bony lesions. It is also
no evidence that home oxygen (choice C) is an effective more expensive than plain radiographs. When search-
treatment to reduce the occurrence of sickle cell crises. ing for metastatic cancer in other sites, a CT of the
Home oxygen therapy has a very limited role that chest, abdomen, and pelvis is often ordered. It is not
focuses on patients with severe COPD. It is not used in necessary, however, when looking for osteolytic lesions
the management of any form of anemia. from multiple myeloma.
In most patients, sickle cell pain is not chronic, but Similar to CT scan, MRI (choice C) can provide a great
rather is characterized by an acute pain crisis caused by deal of information about internal organs. As MRI
a vaso-occlusive chain reaction of sickling. As such, requires more time, is more costly, and does not pro-
most patients do not need long-term outpatient nar- vide much more information than radiographs, it is
cotics, such as transdermal fentanyl (choice E). Narcotic generally a second-line test. For patients with bony pain
therapy, however, is a cornerstone of the acute treat- who have normal roentgenograms, MRI may be an
ment of a sickle crisis, which can be extremely painful. appropriate next test.

46
Internal Medicine Answers and Explanations

PET scan (choice D) and technetium-99m bone scans of COPD treatment. Ipratropium is generally more
(choice E) are not useful for these lesions. Both scans are effective in COPD, though albuterol can provide a
best suited for metabolically active lesions, not the lytic quicker response. Many patients with severe symptoms
lesions of multiple myeloma. The lesions are not caused require both, conveniently available in a combined
by direct tumor infiltration and growth, but rather by inhaler. Neither provides a survival benefit.
hyperactive osteoclasts. As such, these lesions are “cold” Pulmonary rehabilitation (choice D) can improve quali-
and will not light up on these scans. An example of an ty of life through a program of exercise, breath training,
osteoblastic lesion that would show up well on either and psychosocial support. It does not improve mortality.
scan is metastatic prostate cancer, which also can present Other interventions that may improve symptoms
with pathologic fractures and bony pain. include lung reduction surgery and tracheostomy to
reduce physiologic dead space.
31. The correct answer is A. This patient has acute cholan-
gitis most likely caused by a stone in the common bile Steroid inhalers (choice E) only benefit a small percent-
duct. The initial treatment for acute cholangitis focuses age of patients with COPD and should not be a routine
on stabilizing the patient with intravenous fluids and component of COPD management (though they are).
antibiotics that provide adequate gram-negative and Parenteral steroids, however, are indispensable in the
anaerobic coverage. Approximately 80% of patients treatment of an acute exacerbation. These two different
respond to this treatment alone, eventually clearing the uses of steroids in the treatment of COPD should not
obstruction or stabilizing to the point that elective bil- be confused.
iary drainage can be arranged. For patients who do not
33. The correct answer is B. This patient has temporal
improve within 24 hours of treatment, emergent biliary
arteritis and is now developing visual changes. The
drainage needs to be arranged.
most pressing issue is to start steroids, the main treat-
Endoscopic ultrasound (choice B) has an excellent sen- ment for temporal arteritis. The pretest probability of
sitivity for detecting common bile duct lesions. It is not this patient having temporal arteritis is extremely high
a therapeutic intervention, however, and will not suc- given her physical findings and laboratory studies con-
cessfully treat this patient’s condition, though it may be sistent with significant inflammation. As such, steroids
useful if ERCP and other imaging studies fail to delin- can be started even before a temporal artery biopsy
eate the pathology of this patient’s obstruction. (choice E) is obtained. Steroids on occasion may reduce
Endoscopic retrograde cholangiopancreatography the amount of inflammation seen on biopsy, but usual-
(ERCP; choice C) does not need to be performed at ly not to the point at which diagnosis is hindered. An
3 AM and should not be started until the patient is sta- outpatient referral for a biopsy is not a reason to delay
bilized. There is no need to wake the GI fellow. Rather, starting treatment.
admit the patient for fluids and antibiotics, and call GI Broad-spectrum antibiotics (choice A) and a lumbar
in the morning. puncture for CSF analysis (choice C) would be appro-
If this patient continues to have pain and ERCP is not priate if meningitis were suspected. This patient’s con-
successful, surgical decompression and treatment with stellation of symptoms, including unilateral visual
a T-tube (choice D) or exploratory laparotomy (choice impairment, temporal artery tenderness, and signs of
E) may be necessary. Neither, however, is the first step in systemic inflammation (e.g., fever, elevated sedimenta-
the treatment of this patient. Remember, most patients tion rate) are much more consistent with temporal
with ascending cholangitis improve with fluids and arteritis than meningitis.
antibiotics. An MRI of the brain (choice D) is unlikely to reveal any
abnormalities in this patient, as the vasculitis is
32. The correct answer is B. Only two interventions have
extracranial and does not involve the brain parenchy-
been shown to improve mortality in patients with
ma. Some centers are using ultrasound to image the
COPD: smoking cessation and continuous oxygen.
temporal arteries in lieu of biopsy, though this is still
Moreover, continuous oxygen therapy has been proven
controversial.
beneficial only in patients with baseline hypoxemia,
specifically a PaO2 of less than 55 mm Hg, or a satura- 34. The correct answer is E. Gastric cancer can present as a
tion of less than 88%. This patient has severe COPD bleeding or nonhealing gastric ulcer. Biopsy is essential
and would benefit from home oxygen therapy. to rule out malignancy. In general, it is recommended
Albuterol (choice A) and ipratropium (choice C) pro- that, if safe, the ulcer be biopsied during the initial
vide symptomatic relief and are essential components endoscopy. Eight to 12 weeks later, a repeat endoscopy

47
USMLE Step 3 Assessment Exam

and biopsy should be performed to determine if the A D-dimer level (choice C) can be used to rule out pul-
ulcer has healed with acid suppression therapy and to monary embolus in low-risk patients. A positive D-dimer,
increase the yield from more tissue samples. It is hoped however, adds little to the management of high-risk
this patient has bled because of his alcohol and NSAID patients and is not necessary for this man’s diagnosis.
use, rather than from a malignant gastric ulcer. Pulmonary angiography (choice D) is the gold standard
A barium swallow study (choice A) can show ulcer for diagnosing pulmonary emboli. As the procedure is
craters, though the sensitivity is much lower than invasive, has a high degree of mortality and morbidity,
endoscopy. Further, it is impossible to obtain tissue and is not always readily available, however, it is not a
samples with a barium study, essential to determine if recommended first choice in diagnosing a pulmonary
an ulcer is malignant or not. As such, endoscopy, not embolus.
barium, is recommended as the primary tool to evalu- A ventilation/perfusion scan (choice E) is an excellent
ate gastric ulcers. tool to diagnose a pulmonary embolus, but is often
A CT scan of the chest, abdomen, and pelvis (choice B) indeterminable in patients with pre-existing lung dis-
is not appropriate at this time. If this patient is deter- ease. This patient has a history of silicosis and chronic
mined to have a malignant ulcer, then a “pan-scan” is bronchitis; this study likely will come back as indeter-
appropriate to stage the malignancy. minate. A CT angiography can allow examination of
Although this patient should be followed clinically the vessels for filling defects, even if pre-existing
(choice C), further testing, specifically an upper parenchymal lung disease is present.
endoscopy, is essential regardless of his symptoms.
36. The correct answer is C. This patient has systemic scle-
Early gastric cancer may have a subtle presentation. By
rosis. Her physical examination is classic for diffuse,
the time symptoms are prominent, the disease may
systemic disease rather than a more limited CREST syn-
have progressed significantly.
drome. Her dyspnea is worrisome for pulmonary
Helicobacter pylori testing (choice D) does not need to hypertension, her GERD is classic for the condition
be repeated. Assays used for H. pylori are extremely sen- and, most worrisome, the new-onset hypertension rais-
sitive. Further, antibody tests (as opposed to breath ure- es questions of sclerodermal renal crisis, a common
ase) stay positive after exposure, resulting in a high cause of death in these patients. Her serologies are clas-
false-positive rate. It is unlikely, then, that this patient’s sic for the disease. Rheumatoid factor is nonspecific for
test represents a false-negative result. rheumatoid arthritis and is often present in systemic
sclerosis, as are antinuclear antibodies. Anticentromere
35. The correct answer is B. This patient has a classic pre-
antibodies strongly suggest CREST syndrome, whereas
sentation of a pulmonary embolus after a period of
the highly specific but not sensitive anti-SCL-70 highly
immobilization (a 2-day bus ride). All of the listed
correlates with systemic sclerosis. This patient’s hyper-
studies may be useful in the workup of a suspected
tension, elevated creatinine, and proteinuria indicate
pulmonary embolus. With this patient’s pre-existing
the early stages of sclerodermal renal crisis, which is
lung disease, however, the most appropriate diagnostic
treated with an ACE inhibitor.
study is CT angiography, also known as a spiral CT. CT
angiography, which also often includes in the protocol Hydrochlorothiazide (choice A) is not indicated for this
a scan of the lower extremities, has a sensitivity of more patient’s hypertension. An ACE inhibitor can reduce
than 90%. Further, missed emboli are often small and glomerular damage from altered filtration pressures
may not have the same clinical consequences (i.e., the caused by sclerosis and narrowing of the renal arteries.
missed 1 in 10 may not carry the same mortality as the Further, given this patient’s acute decline in renal func-
average untreated pulmonary embolus). As the proce- tion, HCTZ is unlikely to be particularly effective; the
dure requires a significant infusion of potentially agent usually loses effect as its filtration into the renal
nephrotoxic contrast dye, it is not appropriate for tubule declines.
patients with renal disease. Ibuprofen (choice B) is not indicated for the treatment
A chest radiograph (choice A) is not a sensitive or spe- of systemic sclerosis and, in general, should be avoided
cific study to diagnose a pulmonary embolus. Rarely an in patients with impending renal failure, particularly if
area of hyperlucency (Westermark sign), pulmonary diminished glomerular perfusion is believed to be the
infarction (Hampton hump), or large pulmonary underlying pathogenesis of their renal failure.
artery may be seen. The most common radiographic Methotrexate (choice D) is a disease-modifying agent
manifestation of a pulmonary embolus is nonspecific commonly used to treat rheumatoid arthritis. It is not
atelectasis. used in the treatment of systemic sclerosis.

48
Internal Medicine Answers and Explanations

Prednisone (choice E) is used for a variety of condi- This patient does not need to be ruled out for a myocar-
tions, including lupus flares. This patient has systemic dial infarction (choice C). This is a chronic problem, and
sclerosis, one of many antinuclear antibody-positive the demand ischemia is entirely exertional. There is noth-
conditions other than lupus. ing to suggest that this patient’s pain is something new.
A stress echocardiogram or nuclear perfusion study
37. The correct answer is D. Statistically, papillary carcinoma
(choice D) is useful in evaluating patients with unstable
is the most likely diagnosis, as this type of malignancy
angina to see if there are areas that may be amenable to
accounts for 60 to 70% of all thyroid carcinomas. Further,
coronary reperfusion techniques, such as stenting or
papillary carcinoma is associated with calcifications seen
bypass. This patient has stable angina and will not ben-
on radiograph and with a history of radiation exposure. If
efit from evaluation for reperfusion at this time.
this patient’s fine needle aspiration reveals this diagnosis,
surgical resection is the most appropriate treatment for A stress electrocardiogram (choice E) can be used to
an isolated, nonmetastatic papillary carcinoma. evaluate patients with suspected coronary artery dis-
ease. This patient already has angina, and thus little is to
Anaplastic carcinoma (choice A) is rare, accounting for
be gained from an outpatient stress electrocardiogram.
at most 10% of thyroid malignancies. It is also more
common in females and elderly patients. Classically, the 39. The correct answer is E. This patient is supratherapeu-
lesion enlarges quickly and painfully. Most patients die tic on his warfarin dose to dangerous levels that have
within a year of diagnosis. likely precipitated his gastrointestinal bleed (likely gas-
Follicular carcinoma (choice B) accounts for 15% of tritis caused by an NSAID, though only an endoscopy
thyroid cancers and is also more common in elderly will answer this question). In addition to stopping his
and female patients. Statistically this patient is more warfarin, there are two ways to reverse the coagulopa-
likely to have papillary carcinoma, particularly given thy. First, the patient can be given fresh frozen plasma
the radiographic findings and the history of radiation that contains the missing endogenous factors inhibited
exposure. by warfarin. This will rapidly reverse the coagulopathy
Medullary carcinoma (choice C) is rare, accounting for and is useful if the patient continues to bleed. Second,
5% of thyroid carcinomas. The tumor is metabolically the patient can be given vitamin K, which can compet-
active, producing calcitonin. An additional clue to the itively overcome the effects of warfarin. Vitamin K takes
presence of medullary carcinoma is a family history of longer to act, as the liver has to synthesize these clotting
the disease or the presence of pheochromocytoma, factors (II, VII, IX, X), but lasts considerably longer.
parathyroid hyperplasia, or neuromas, as this condition Cryoprecipitate (choice A) is high in some specific clot-
often occurs as part of multiple endocrine neoplasia ting factors and is useful for patients who receive large-
syndromes. volume transfusions or who have specific factor defi-
Thyroid lymphoma (choice E) is an extremely rare ciencies. It is not necessary to reverse warfarin toxicity.
form of non-Hodgkin lymphoma. It occurs mainly in DDAVP (choice B), or desmopressin, is used mainly for
elderly patients, particularly in those with a long-stand- two functions. First, it induces endothelial release of
ing history of Hashimoto thyroiditis. This entity repre- von Willebrand protein, and thus can be used to treat
sents less than 1 to 2% of thyroid tumors. von Willebrand disease. Second, it can reverse some
platelet dysfunction caused by uremia, and is thus a
38. The correct answer is B. This patient has stable angina treatment, together with dialysis, for uremia-induced
and has not been taking his medications as prescribed. platelet dysfunction.
As the pattern is well established and the pain is not
A platelet transfusion (choice C) is not necessary for
new in frequency or intensity and resolves with rest,
this patient’s borderline reduced platelets. In patients
further workup is not warranted. Patients with coro-
who are actively bleeding with thrombocytopenia,
nary artery disease gain a proven survival benefit from
platelet levels are generally kept greater than
81 mg of aspirin daily, however, which should be added
50,000/mm3, though there are no absolute transfusion
to this patient’s regimen. Beta-blockers and, for patients
thresholds.
with reduced ejection fractions, ACE inhibitors also can
improve survival. Protamine (choice D) can reverse heparin, not warfarin.
This patient’s high PT/INR reflects his supratherapeutic
This patient does not need a cardiac catheterization
warfarin levels, likely caused by a combination of poor
(choice A). The lack of ongoing pain, the chronicity of
vitamin K intake, antibiotic use, and drug interactions.
the problem, and the absence of EKG changes argue
against urgent intervention.

49
USMLE Step 3 Assessment Exam

40. The correct answer is D. Polycythemia vera (PV) is a Further clues specific to HSV infection include tempo-
chronic myeloproliferative disorder that can result in an ral lobe hemorrhage on CT scan and severe derange-
elevation of all cell lines, but mainly in myeloid cell ments in behavior.
lines. It is a primary disorder, as opposed to a secondary Dexamethasone (choice C) is increasingly being used in
erythrocytosis caused by chronic hypoxic states. This the early treatment of meningitis. It has been proven to
patient could have PV, but may also have a reactive ery- reduce post-meningitis neurologic complications, par-
throcytosis caused by chronic hypoxia (sleep apnea, ticularly in streptococcal infections. It does not have a
tobacco history). If this is a reactive increase in hemat- proven survival advantage, however, and would not have
ocrit, one would expect an elevated erythropoietin changed this patient’s course. Appropriate antibiotics
level, as this is the autoregulatory modulator of would have improved her chances of survival, though
increased red blood cells. Additionally, if this patient’s the low CSF glucose is a poor prognostic indicator.
condition is caused by a renal cell carcinoma, which
Piperacillin/tazobactam (choice D) is a broad-spectrum
may present with increased hematocrit, one also would
antibiotic, but is not recommended for the treatment of
expect elevated erythropoietin levels. Incidentally, the
meningitis. Specific third-generation cephalosporins,
presentation of pruritus after a shower is classic for PV,
such as ceftriaxone and cefotaxime, provide better cen-
as is a constellation of facial plethora, splenomegaly,
tral nervous system penetration and provide coverage
and erythromelalgia, or burning in the hands and feet,
for most common causes of meningitis.
believed to represent microvascular occlusive disease
from sludging of the blood. Vancomycin (choice E) is not a great agent to treat
meningitis, as it has poor central nervous system pene-
This condition is characterized by high, not low, blood
tration and is a bacteriostatic, not bacteriocidal, agent.
viscosity and red cell mass (choice A). The increased
It is often used, however, as empiric treatment in
blood viscosity from an absolute erythrocytosis results
patients at risk for methicillin-resistant Staphylococcus
in an increased risk for thrombotic events.
aureus infections. If this patient had had a recent neu-
Also in the differential diagnosis of this patient is rosurgical procedure or had come from a nursing home
chronic myelocytic leukemia (CML), which may pre- or hospital, vancomycin would be appropriate empiric
sent with similar abnormalities on a blood smear. CML treatment. Vancomycin, however, has little (though
has a low leukocyte alkaline phosphatase level (choice some) action against Listeria. The recommended agent
B), whereas PV has a high level (representing red cell is ampicillin.
turnover), making this a useful test to differentiate the
two conditions. Similarly, the cyanocobalamin, or vita- 42. The correct answer is B. This patient has hyponatremia
min B12, level (choice C) is elevated also, reflecting red caused by SIADH (syndrome of inappropriate antidi-
cell turnover and abnormal erythropoiesis. uretic hormone), likely from a small cell lung cancer.
Often all cell lines are elevated, including white blood The most appropriate initial treatment for SIADH is
cell and platelet counts (choice E), though the predom- fluid restriction, though occasionally a loop diuretic
inant abnormality is an absolute erythrocytosis. (which loses relatively more water than sodium) or
hypertonic saline is needed. In general, when evaluating
41. The correct answer is B. This patient has Listeria mono- hyponatremia, fluid restriction is always an appropriate
cytogenes meningoencephalitis. This pathogen should initial treatment until the cause is known. SIADH does
be considered and empirically covered in neonates, not respond to fluids, which paradoxically may make the
elderly patients, or immunosuppressed patients, such as situation worse.
those on steroids (this patient) or those with HIV infec- Demeclocycline (choice A) is rarely used to treat chronic
tion. Listeria meningoencephalitis often presents with SIADH. It works by inducing a nephrogenic diabetes
seizures. Listeria is a small, gram-positive rod, a specif- insipidus, rendering the renal tubule relatively insensitive
ic though highly insensitive Gram stain result. to ADH. It is not an appropriate initial treatment.
Cephalosporins have no activity against Listeria; rec-
Hydrochlorothiazide (choice C) and other thiazide
ommended treatments include high-dose penicillin or
diuretics are contraindicated in hyponatremia. They
ampicillin, though some agents (such as the carbapen-
function by inhibiting the distal convoluted tubule reab-
ems) have activity against the organism.
sorption of sodium and chloride. This portion of the
Acyclovir (choice A) can be used for treatment of sus- kidney serves to dilute urine. By inhibiting this function,
pected herpes meningoencephalitis. Viral meningitis or this drug results in relatively salt-concentrated urine,
meningoencephalitis usually presents with a lymphocyt- and thus can induce hyponatremia.
ic predominance on cerebrospinal fluid (CSF) analysis.

50
Internal Medicine Answers and Explanations

Hypertonic saline (choice D) is needed only in emer- reversible cause of dementia caused by a relative over-
gencies. If this patient were obtunded or seizing, then abundance of CSF fluid with resultant distention of the
initial correction to approximately 120 mEq/L would be ventricles and compression of the limbic system. The
appropriate, together with fluid restriction. clinical presentation is one of moderate to severe
Normal saline (choice E) should not be given to dementia (though often less so than in Alzheimer
patients with SIADH. The antidiuretic hormone results dementia), a wide-based gait, and urinary inconti-
in reabsorption of the free water, whereas the 154 nence. The treatment of choice is a CSF shunt or
mEq/L of sodium is simply wasted in the urine. This repeated large volume lumbar punctures.
can result in a paradoxic worsening of the patient’s Often there is some overlap in presentation with
hyponatremia. Parkinson disease, and patients may be started inappro-
priately on carbidopa/levodopa (choice A). The CT
43. The correct answer is B. This patient has sarcoid, a con- findings and urinary incontinence, together with a
dition significantly more common in African American wide-based gait without true weakness or ataxia, argue
women. The disease is characterized by an inappropriate in favor of a diagnosis of NPH, which will not improve
T-cell response to self-antigens, resulting in the forma- with this treatment.
tion of noncaseating granulomas. Classic findings
Vitamin E and donepezil (choice B) are two treatments
include hilar lymphadenopathy in a young, African
that have shown promise in the treatment of Alzheimer
American woman, erythema nodosum, and elevated
dementia. Both can delay the rate of cognitive decline
ACE levels (present in 75% of patients with sarcoid).
and delay the need for a nursing home. Neither is used
Occasionally, a large granuloma burden will result in
to definitively treat NPH.
hypercalcemia. This is caused by granuloma production
of active vitamin D, or 1,25-dihydroxy cholecalciferol, Only approximately 21% of patients improve with
which results in increased calcium absorption by the gut shunting. Given the potential to reverse the dementia,
(its main mechanism). however, further treatment (choice C) should be offered.
Acid-fast bacillus (choice A) would indicate TB or atyp- Patients with NPH often have a mild hyponatremia,
ical mycobacterium in this setting. This patient has believed to be caused by compression of the posterior
nothing to suggest TB exposure and recently had a neg- pituitary, resulting in release of ADH. This patient’s
ative PPD. sodium is not low enough to explain her symptoms and
does not need to be replaced to normal (choice E).
Granulomas with central necrosis (choice C) are seen
commonly in tuberculosis. Sarcoid is characterized by 45. The correct answer is A. This patient likely has hepatic
noncaseating granulomas. The final diagnosis of the encephalopathy and needs evaluation for spontaneous
condition rests on histopathologic confirmation, which bacterial peritonitis, which can exacerbate or initiate
requires lymph node or parotid biopsy. If there are no the condition in a patient with previously compensated
peripheral sites of active disease, biopsy by way of bron- liver disease. Patients often have asterixis, though it may
choscopy or mediastinoscopy is necessary. be difficult to get an encephalopathic patient to extend
Sarcoid is a disease of T-cell dysfunction, and patients the wrists. Other neurologic findings may include rigid-
are somewhat immunocompromised. In most patients, ity, not flaccidity (choice C), hyperreflexia, not hypore-
however, CD4 cell counts (choice D) are normal. A flexia (choice D), and focal neurologic findings such as
reduction in CD4+ T cells is consistent with HIV, hemiplegia.
though up to 10% of patients with sarcoid have a gen- Fetor hepaticus (choice B) correlates with severe end-
eral lymphopenia. stage liver disease but is not usually present in cases of
Skin biopsy (choice E) is unlikely to reveal noncaseating hepatic encephalopathy. Fetor hepaticus is a foul, musty
granulomas. This patient has erythema nodosum, a smell associated with the presence of mercaptans in the
nonspecific nodular rash commonly appearing on the blood.
tibias. It is believed to represent a delayed hypersensi- Nystagmus (choice E) is associated with Wernicke-
tivity reaction and is actually a good prognostic indica- Korsakoff syndrome, which this patient is certainly at
tor, as it shows a healthy host immune response. These risk for given his history of alcoholism. The patient’s
lesions are caused by this reaction, not by direct infil- quick response to lactulose is classic for hepatic
tration with granuloma. encephalopathy, however, which is likely his primary
diagnosis. The question now is what triggered his
44. The correct answer is D. This patient likely has normal
encephalopathy. Although many patients will not have
pressure hydrocephalus (NPH), a rare but potentially

51
USMLE Step 3 Assessment Exam

a clear cause for their decompensation, many others will Low complement levels (choice B) indicate active
have an underlying infection, such as an early pneumo- lupus, but occur in a variety of conditions. A host of
nia, a urinary tract infection, or spontaneous bacterial inflammatory, autoimmune, and infectious conditions
peritonitis. Dietary and medical compliance are also activate the complement pathway, resulting in con-
common precipitants. Failure to take outpatient lactu- sumption of complement components and low mea-
lose, a sudden increase in protein consumption or sured levels.
dehydration (which may be caused, ironically, by too A positive ANA (choice C) is extremely sensitive, not
much lactulose) can induce hepatic encephalopathy. specific. It is negative in less than 3% of cases of SLE,
but is positive in up to 10% of normal patients.
46. The correct answer is B. TSH is the single most useful
test when evaluating hypo- (as in this patient’s case) or Positive anti-RNP (choice D) is seen most commonly in
hyperthyroidism. It is also used to monitor a patient’s mixed connective tissue disorder, but may be positive if
response to treatment, as changes in the TSH are present in more than half of patients with SLE.
believed to be more indicative of a response to treat-
48. The correct answer is E. The standard of care for the
ment than changes in the free T4.
treatment of a single episode of pulmonary embolus is
The free T4 index (choice A) is an old test used to indi- 3 to 6 months, preferably the latter, with warfarin. Only
rectly measure free T4. It is no longer used, now that patients who fail warfarin therapy (i.e., those who have
assays to measure free T4 directly are ubiquitous. a recurrent clot) or have a contraindication to antico-
Free T4 (choice C) is a useful test to gauge the extent of agulation require treatment with a mechanical filter,
hypo- or hyperthyroidism. TSH is considered the most such as a Greenfield filter (choice B).
useful test of determining if a patient is euthyroid, how- Antiplatelet agents such as aspirin or clopidogrel
ever, and is the most useful test for monitoring a (choice A) are inappropriate for the management of a
response to treatment with supplemental thyroxine. pulmonary embolus. The coagulation cascade needs to
Total T4 (choice D) is a test that is not used often. A vari- be interrupted with agents such as warfarin, heparin,
ety of conditions can alter levels of thyroid-binding enoxaparin, and newer direct thrombin inhibitors.
globulin, which causes a fluctuation in the total T4 level. Heparin (choice C) is an appropriate immediate treat-
The free (unbound) T4, however, which is metabolically ment for a pulmonary embolus, as it rapidly reaches ther-
active, is maintained at homeostatic levels that may not apeutic levels compared with oral warfarin. It is difficult
correlate with the total T4 levels. to administer, however, and requires frequent monitoring
Free T3 (choice E) rarely needs to be ordered in the of the activated partial thromboplastin time. As such, it is
workup of hypo- or hyperthyroidism. T3 levels are reg- not an ideal agent for long-term treatment. Patients who
ulated mainly at the tissue level and are much less likely have a contraindication to warfarin, such as those who are
to be abnormal than are T4 levels. pregnant, may require warfarin or enoxaparin.
Enoxaparin (choice D) is a subcutaneously administered
47. The correct answer is E. Positive anti-Smith and anti-
low-molecular weight heparin. It rapidly reaches thera-
dsDNA antibodies are very specific for systemic lupus
peutic levels, usually does not require monitoring, and
erythematosus (SLE). Unlike a positive ANA that may
can be self-administered by patients. Warfarin, however,
be present in a variety of autoimmune diseases and in
is considered to be cheaper, easier, and has a longer track
many unaffected people, anti-Smith and anti-dsDNA
record in treating a pulmonary embolus. As such, enoxa-
rarely are seen in conditions other than SLE. As such,
parin usually is used as a bridge to warfarin treatment. It
this is a good test to rule in the condition, though many
is started together with warfarin and then stopped once
patients with SLE may have the condition without hav-
the INR reaches therapeutic levels (an INR of 2 to 3).
ing these antibodies present. The other choices listed
are not specific to SLE. 49. The correct answer is D. Among the listed agents,
This patient’s rash (choice A) is not specific for SLE. spironolactone is the only one to show a survival bene-
The rash described is most consistent with a discoid fit. The mechanism is believed to be reduced cardiac
rash, as it involves mainly the scalp and ears, rather than fibrosis and remodeling, by way of modulation of the
appearing as an erythematous band across the malar renin-angiotensin-aldosterone system. Low daily doses
areas. Most patients with discoid lupus do not progress are adequate to see this effect. Other agents that improve
to SLE, and most patients with SLE (approximately survival are aspirin, beta-blockers (usually metoprolol
75%) do not have this rash. or carvedilol), ACE inhibitors, and the combination of
hydralazine and nitrates.

52
Internal Medicine Answers and Explanations

Digoxin (choice A) has been studied extensively, but has the biopsy should be arranged for as soon as possible
never imparted a survival advantage. It can result in rather than waiting (choice B).
improved symptoms and fewer hospitalizations, but A CT scan of the abdomen and pelvis (choice A) is not
also can be extremely toxic, having a narrow therapeu- indicated at this time. If this patient has a positive biopsy,
tic window. a CT scan is often part of the staging workup, though it
Furosemide (choice B) improves symptoms, and, like has questionable sensitivity. Similarly, a radionucleotide
fluid and salt restriction, is essential to managing con- bone scan (choice C) is indicated only if there is suspicion
gestive heart failure. Loop diuretics do not, however, of metastatic disease; a tissue diagnosis of prostate cancer
improve survival. is necessary before progressing to this test.
Hydrochlorothiazide (choice C) is used occasionally in A transrectal ultrasound (choice E) is not necessary as
difficult to treat edema. The combination of part of the workup for a prostatic nodule, though it
hydrochlorothiazide and furosemide (or any other loop may be used to guide a biopsy. Because the consistency
diuretic) serves to synergistically enhance diuresis. of a cancerous nodule varies, it may be hypo-, hyper-,
Although this may help manage otherwise refractory or isoechoic, resulting in a substantial number of false
edema, it has not been shown to improve survival. negatives. Regardless of the result of transrectal ultra-
Alpha-blockers, such as terazosin (choice E), are no sonography, the patient will require a biopsy.
longer used to treat congestive heart failure. Afterload
52. The correct answer is D. This patient likely has sponta-
reduction is better achieved with hydralazine or an
neous bacterial peritonitis (SBP), a common infection
ACE inhibitor, agents that have been proven to
in patients with long-standing ascites. Patients sponta-
improve survival.
neously seed their peritoneum, leading to an often
50. The correct answer is C. This patient has a classic pre- indolent infection that is caused by a single organism.
sentation of an asymptomatic case of lupus anticoagu- The most common organisms are E. coli,
lant. This common condition can predispose patients Enterobacteriaceae, and S. pneumoniae. Given the vari-
to recurrent thrombotic (not hemorrhagic) events. If ability in clinical presentation, the only way to make the
the patient is asymptomatic, further treatment is not diagnosis is by paracentesis. A positive paracentesis is
necessary. Classic clues include a false positive VDRL one with greater than 500 total WBCs, of which greater
for syphilis (but a negative specific test such as an FTA- than 250 cells are neutrophils. Organisms are rarely
ABS or dark-field spectroscopy), an elevated PTT, and, seen on the Gram stain, and the presence of a large
if symptomatic, a history of spontaneous abortions or organism burden, particularly if polymicrobial, should
recurrent venous clots. prompt one to think of a perforated viscus.
Vitamin K (choice A) and fresh frozen plasma (choice Additional diuretics, such as furosemide (choice A), may
B) play no role in the management of this patient’s con- be necessary, but should not be started until it is clear
dition. She is not suffering from vitamin K deficiency or what is causing this patient’s worsening symptoms.
inhibition, which would result in an elevated PT/INR, Cefotaxime (choice B) is the treatment of choice for
rather than an elevated PTT, and she does not have a SBP. A paracentesis should be performed before treat-
deficiency of factors that can be treated with plasma. ment, however, to determine if the infection is indeed
Warfarin should be used for long-term management of present. Unlike bacterial meningitis, delaying antibi-
patients with this condition who develop thrombotic otics by an hour or two will not increase the risk for
events. Unlike the usual treatment for deep venous poor disease outcomes.
thrombosis, which is a goal INR of 2 to 3 seconds An abdominal CT scan or ultrasound (choice C) may
(choice D), these patients need more powerful antico- be necessary in this patient’s management, particularly
agulation. The goal INR is usually set higher, at 2.5 to if SBP is not the cause of his worsening symptoms. A
3.5 seconds (choice E), though individual institutions sudden increase in ascites may indicate a serious com-
may vary. plication of cirrhosis, such as the development of hep-
atocellular carcinoma or a clot in the hepatic veins.
51. The correct answer is D. Prostate nodules and asym- The first test for this patient, however, should be a
metric prostates need to be biopsied, particularly in diagnostic paracentesis.
patients at risk for prostate cancer, such as this patient.
A thoracentesis (choice E) is not necessary. Many
Even if the PSA level is low, this patient, based on
patients with liver disease and ascites have right-sided
pretest probability and the poor specificity and sensitiv-
effusions, as fluid easily moves through small pores in
ity of the PSA test, will need a prostate biopsy. As such,

53
USMLE Step 3 Assessment Exam

the diaphragm above the inflamed liver. If there is 55. The correct answer is B. This patient has Parkinson
doubt about this patient’s effusion, old radiographs disease. The primary symptoms are resting tremor in
should be obtained to determine if the effusion is the hands, arms, legs, jaw, and face, rigidity or stiffness
chronic, in which case it could be left alone. of the limbs and trunk, bradykinesia, and postural
instability or impaired balance and coordination.
53. The correct answer is E. This patient has a clinical pre- Patients may also have difficulty walking, talking, or
sentation consistent with pyelonephritis. No additional completing other simple tasks. The initial treatment for
tests are needed, though the culture data, once avail- moderate to severe Parkinson disease (such as in this
able, should be used to tailor initial empiric antibiotic case) is dopamine or a dopaminergic agonist, such as
therapy. If the patient fails to improve on appropriate bromocriptine. L-dopa is given with carbidopa, which
treatment, then one needs to consider a complication of inhibits peripheral breakdown of L-dopa.
pyelonephritis or another cause. Likely complications
Amantadine (choice A) has some mild anti-Parkinson
might include perinephric abscess, papillary necrosis,
effects, believed to be caused by increased central
or emphysematous pyelonephritis.
dopamine release. The agent is a distant third to
Ultrasound (choice A) can be used to look for obstruc- dopamine and dopaminergic agonists, however, and
tion and hydronephrosis, and may also show a renal should not be the initial or primary treatment for mod-
abscess. A CT scan (choice B) reveals the same infor- erate to severe cases.
mation and may have a higher sensitivity. At this point,
however, neither test is warranted, as there is no reason Donepezil (choice C) is a cholinesterase inhibitor that
to suspect this patient has something more than a bad may delay the progression of Alzheimer dementia in
case of pyelonephritis. some patients. It is not useful in the treatment of
Parkinson disease.
An IVP (choice C) can be used to look for renal stones
and will reveal the anatomy of the ureter. The test is not Sertraline (choice D) is an SSRI useful in the treatment
appropriate in this patient, however, who has a presen- of pseudodementia, or depression, which can present as
tation more consistent with pyelonephritis than with a patient with near catatonia. The tremor and gait
renal stones. Further, if stones were suspected, ultra- instability are more suggestive of a disease of the basal
sound or noncontrast CT scan are often the first test, as ganglia, however, such as Parkinson disease.
these modalities avoid potentially nephrotoxic dye. Vitamin B12 and folate (choice E) would be useful if this
A KUB (choice D) is generally low yield, though it may patient had a deficiency of either. Vitamin B12 deficiency
show emphysematous pyelonephritis or emphysematous in particular can result in a dementia with an abnormal
cystitis, if present. It is generally a useful test for evaluat- gait caused by posterolateral spinal cord degeneration. A
ing for bowel obstruction or perforation and will often macrocytic anemia, however, would be expected on lab-
pick up renal stones. It is not indicated in this patient, oratory values. Further, the bradykinesia, lack of facial
however, with a classic presentation of pyelonephritis. expression, and resting tremor are classic for Parkinson
disease.
54. The correct answer is D. A positive PPD in a healthcare
worker is an area of induration of greater than 10 mm, 56. The correct answer is A. All patients with long-standing
read 48 to 72 hours after placement of the skin test. cirrhosis, particularly if from chronic hepatitis B or C,
Erythema is also often present, and may be quite signif- should be screened periodically for the development of a
icant. The induration, not the erythema (which may hepatoma. Screening strategies include the use of period-
have quite a large diameter), is what needs to be read ic ultrasounds and laboratory evaluation for the presence
when interpreting a PPD. of serologic tumor markers. Additionally, in patients
with cirrhosis who suddenly develop a deterioration or
The area of erythema, whether 5 mm (choice A), 10
increased ascites, the development of a hepatoma needs
mm (choice B), or 15 mm (choice E) is irrelevant. The
to be considered, in addition to spontaneous bacterial
induration is used to determine a positive PPD, as the
erythema may be quite variable and nonspecific. If peritonitis and clots in the inferior vena cava or hepatic
there is any confusion, the lesion should be palpated or veins. AFP is a tumor marker that is markedly elevated
marked with a ballpoint pen to determine the extent of in patients with hepatoma or a germ cell tumor.
the raised area. Although the marker is nonspecific, high elevations are
more likely to be associated with these tumors than with
An area of induration greater than 5 mm (choice C) is other cancers.
considered a positive test in patients who are immuno-
compromised (mainly HIV patients) and those with Beta-hCG (choice B) is elevated in some cases of testic-
close contacts who have active tuberculosis. ular cancer and in gestational trophoblastic disease

54
Internal Medicine Answers and Explanations

(which includes hydatidiform mole, choriocarcinoma, NPH insulin often peaks 6 to 8 hours after administra-
and other placental site trophoblastic tumors) and tion. As such, increasing the dose of morning NPH
ectopic or intrauterine pregnancy. insulin (choice D) is unlikely to help control this
CA-125 (choice C) is a tumor marker often seen in ovar- patient’s blood glucose levels.
ian cancer that is used occasionally as a screening test in Increasing the dose of regular insulin before breakfast
high-risk populations, though it still has a low positive (choice E) will help control her morning hyperglycemia.
predictive value. It is not associated with hepatoma. It does not address the likely nocturnal hypoglycemia,
CEA (choice D) is elevated in a variety of gastrointesti- however, or the resultant hyperglycemia that is likely
nal malignancies and is generally too nonspecific to be occurring for hours before she awakes.
used for tumor screening, even in high-risk popula-
58. The correct answer is A. Patients with severe rheuma-
tions. It is useful, however, in patients with known
toid arthritis may have involvement of C1 and C2,
treated malignancies (particularly colon cancer) who
resulting in atlantoaxial subluxation. Hyperextension,
need to be monitored closely for evidence of tumor
such as that during intubation, can result in spinal cord
recurrence. It is not strongly associated with hepatoma.
injury. Patients need a screening radiograph or, in an
PSA (choice E) is a tumor marker often seen in prostate emergency, airway management by an expert (anesthe-
cancer that is often used as a screening test. It is not siologist, preferably with a fiberoptic intubation kit).
associated with hepatoma. The patient’s pattern of injury, involving all of her
extremities and compromising her ability to breathe
57. The correct answer is A. Morning hyperglycemia in
spontaneously, is highly suggestive of a high cervical
patients with diabetes needs close evaluation. Two dif-
spinal injury.
ferent causes of morning hyperglycemia need to be dif-
ferentiated, as they require opposite treatments. Choose A cervical epidural hematoma (choice B) could conceiv-
the wrong one and you can kill your diabetic patient! ably cause similar symptoms. Without a pre-existing
The Somogyi effect is rebound hyperglycemia caused reason to bleed into her spinal cord (an exceedingly rare
by counter-regulatory cortisol release in response to event), however, she is unlikely to do so, even after
nocturnal hypoglycemia. This occurs mainly when receiving a thrombolytic agent.
patients or physicians provide too much of a medium Hypoxic brain injury (choice C) and intracerebral hem-
(such as NPH) or long-acting insulin, which peaks orrhage (choice D) are unlikely to create this pattern of
many hours after administration. These patients need quadriplegia. More likely would be either global deficits
less of their evening dose of long or medium-acting (including cognition) or focal neurologic findings.
insulin. The alternative and much more common cause Prednisone (choice E) has a variety of long-term side
of pre-breakfast hyperglycemia is the Dawn phe- effects, including osteopenia. Although it may con-
nomenon. Regular circadian release of cortisol results tribute to spinal fractures from bone demineralization,
in a peak release between the hours of 3 and 8 AM. the most likely cause of this patient’s spinal condition is
Patients are relatively hypoinsulinemic and should a pre-existing atlantoaxial subluxation worsened by an
receive additional medium- or long-acting insulin the emergency intubation.
evening prior. This patient has signs of nocturnal hypo-
glycemia. Her insulin level should be reduced and she 59. The correct answer is D. This patient has Wernicke
should be advised to wake up and check her 3 AM blood encephalopathy, characterized by the triad of ocular
glucose level for a few nights so that her insulin can be abnormalities (classically nystagmus and a lateral rectus
dosed accordingly. palsy), gait ataxia, and encephalopathy. One way to
Decreasing the dose of regular insulin before dinner remember this is that alcoholic patients have a NAC (nys-
(choice B) is unlikely to be beneficial. Regular insulin tagmus, ataxia, confusion) for getting themselves in
peaks quickly. The patient’s late night hypoglycemia trouble. Wernicke encephalopathy may be triggered by
and early morning hyperglycemia cannot be attributed not being given adequate thiamine (a.k.a. vitamin B1)
to her dose of regular insulin. replacement together with glucose. These patients have
a high carbohydrate diet and low thiamine stores. The
Patients with type 1 diabetes should be allowed to have
sudden glucose load can exhaust thiamine reserves, which
snacks (choice C). They often develop an excellent
are consumed as a Krebs cycle cofactor. Extreme Wernicke
sense of when they are suffering from low blood glucose
can result in Korsakoff amnestic syndrome, characterized
levels and usually know what sources and quantities of
by confabulation and anterograde amnesia.
sugar they need to turn things around.

55
USMLE Step 3 Assessment Exam

Calcium (choice A) needs to be replaced in massive Weakness and lethargy, arthralgias, impotence, and
blood transfusions (typically greater than 8 to 10 U of liver function abnormalities are common presenting
PRBCs). Citrate used to preserve blood chelates ionized complaints. Occasionally patients may develop a bronze
calcium. This patient is unlikely to be suffering from hue to their skin, resulting in the “bronze diabetes” of
severe hypocalcemia, which, in any case, would be hemochromatosis. The most effective, simplest, and
unlikely to explain his constellation of syndromes. cheapest way to remove excess iron is with regular phle-
Folate (choice B) deficiency does not explain this patient’s botomy.
symptoms. Folate deficiency can result in a megaloblastic Deferoxamine (choice A) and penicillamine (choice D)
anemia and often contributes to the multifactorial mega- are chelating agents. Although it is possible to chelate
loblastic anemia seen in alcoholic patients. the iron, this treatment is rarely used, as phlebotomy is
Magnesium (choice C) should be given to alcoholic more effective. If patients present with conditions
patients empirically, together with thiamine, folate, and requiring immediate removal of the excess iron, then
a multivitamin, as they are often total body-deficient in deferoxamine and phlebotomy may be appropriate.
magnesium, though their measured blood levels may be Penicillamine is used mainly to bind copper and used to
normal. be a main agent in the treatment of Wilson disease.
Vitamin B12 (choice E) deficiency may be seen in This patient is iron overloaded. Ferrous sulfate (choice
severely malnourished alcoholic patients and can result B) will only worsen the patient’s symptoms.
in gait disturbances and a reversible dementia. Given Liver transplant (choice C) is only necessary in advanced
the acuity of this patient’s symptoms and the character- cases of cirrhosis and hepatocellular carcinoma. There is
istic triad and presentation, however, thiamine defi- no evidence this patient has significant cirrhosis or irre-
ciency is a more likely etiology. versible liver disease.

60. The correct answer is A. Cilostazol is a phosphodi- 62. The correct answer is E. There is no evidence that
esterase inhibitor that causes arterial vasodilatation and screening for lung cancer, even in smokers, reduces the
inhibits platelets. It is a useful treatment for claudica- risk for dying of lung cancer. Some early trials have
tion symptoms, particularly when used in conjunction shown a difference in survival, but this is believed to be
with lifestyle modifications (structured exercise pro- simply because of lead time bias, in which the diagno-
gram and smoking cessation) and other medications, sis was simply made earlier in the normal lifespan of the
such as aspirin, clopidogrel, a statin, and an ACE disease. As such, it is not recommended to screen for
inhibitor, all of which have been shown beneficial in lung cancer at this time.
the treatment of peripheral vascular disease. Radiographs (choice A), cytology (choice C), and com-
Pentoxifylline (choice B) may provide a slight improve- binations of the two (choice B), lack the sensitivity and
ment in claudication symptoms but has been shown to specificity necessary to be effective screening tools.
be less effective than cilostazol in head-to-head trials. CT scans (choice D) may at some point provide ade-
There has never been shown any benefit to increasing quate sensitivity, but currently are not considered cost
the dose of aspirin (choice C) from 81 mg a day (a baby effective and, given the high rate of false positives, actu-
aspirin) to 325 mg a day. Presumably, 81 mg is an ally may be detrimental if they result in unnecessary
adequate dose to inhibit platelets. biopsies.
Clopidogrel (choice D) has been shown to be more
63. The correct answer is C. Chronic hepatitis B should be
beneficial than aspirin in the treatment of claudication
treated, as chronic active disease can progress to cirrho-
symptoms. It should not be stopped in this patient.
sis, hepatoma, or fulminant liver failure. Early treatment
Beta-blockers, such as metoprolol (choice E), were once used interferon monotherapy (choice A), a difficult to
believed to have the potential to worsen peripheral vascu- administer medication with many side effects, including
lar disease. This has never borne out and, given that these significant psychiatric effects. Poorly controlled depres-
patients almost always have significant coronary artery sion or bipolar disorder is a relative contraindication.
disease, there is no reason to withhold a beta-blocker. More recently, lamivudine, originally developed for HIV
treatment, has become the first line agent for treating
61. The correct answer is E. This patient has hereditary
chronic HBV, though its efficacy is similar to interferon.
hemochromatosis, a condition characterized by dysreg-
For patients who fail either, adefovir is a new nucleotide
ulation and increased uptake of iron. Patients often pre-
analog that can be used for patients who do not respond
sent with vague complaints related to iron deposition.
to interferon or who become resistant to lamivudine.

56
Internal Medicine Answers and Explanations

Interferon and ribavirin (choice B) are used in the 66. The correct answer is C. The constellation of findings
treatment of chronic hepatitis C, not HBV. The given in the vignette is good for Cushing syndrome. We
Pegylated version allows for less frequent injections of often think of that diagnosis when shown a picture of a
the medication. patient who exhibits a Cushingoid appearance, but
This patient should receive additional treatment sometimes body habitus is not present in its florid, fully
(choice D), as his prolonged state of infection puts him developed, typical pattern. In that case, the other “coin-
at significant risk for the development of a hepatoma. cidences” (diabetes, osteoporosis, irregular menses, easy
bruisability, and mood swings) should trigger suspicion.
Ribavirin monotherapy (choice E) is never used and is
The other four options given address classic conditions
completely ineffective in the treatment of viral hepatitis.
associated with surgically correctable hypertension, all
64. The correct answer is D. This patient likely has a stru- of which can be seen in young women, but as it happens,
vite stone. Such stones represent 5 to 10% of all kidney each is negated by some of the descriptors.
stones and generally form only in alkaline urine. The In hyperaldosteronism (choice A), serum potassium is
most common culprit is Proteus, which, by way of ure- typically low. Furthermore, hypertension caused by that
ase, creates an alkaline environment. Other common condition does not coincide with diabetes, osteoporo-
causes of urinary tract infections, such as E. coli (choice sis, irregular menses, easy bruisability, or mood swings.
A) and staphylococcal infection (choice E) do not alka- Renovascular hypertension (choice B) is likewise not
linize the urine. associated with the five additional findings listed,
Nephritic syndrome (choice B) and nephrotic syn- whereas it is often suggested by the presence of an
drome (choice C) may or may not be present in this upper abdominal or flank bruit.
woman. Her current urine dipstick findings and hyper- Coarctation of the aorta (choice D), when seen in the
tension are most likely caused by the presence of a uri- adult, often has the classic scalloping under the ribs that
nary tract infection and renal stone (which often creates results from the large collateral vessels that develop over
hematuria) and the resultant pain. She is, however, at the years. As in the other examples given, none of the
risk for diabetic nephropathy, and this may explain the five additional findings present in this patient are seen
dramatic proteinuria. in coarctation. The same could be said for pheochro-
65. The correct answer is B. This patient’s effusions are mocytoma (choice E).
almost certain to be caused by heart failure. Significant 67. The correct answer is A. This patient has subclinical
pulmonary edema often results in bilateral pleural effu- hypothyroidism, defined as an elevated TSH with a nor-
sions, which are transudates. Transudates classically mal T4. Although there is some controversy over whom
appear yellow or straw-colored, are fairly clear, have a to treat, most patients generally benefit from treatment.
protein level that is less than half the serum level, and In patients with cardiac disease or a history of psychiatric
have an LDH level less than 60% of the serum level disorders, the evidence in favor of treatment is strong.
(and less than two thirds the upper limit of normal). Patients are at risk for accelerated atherosclerosis and
Hemorrhagic pleural fluid (choice A) is exudative and recurrent depression if they are not treated (choice C).
should raise concern for malignancy or pleural infarction. Antithyroid peroxidase antibody levels (choice B) may
Pleural fluid with a low glucose and low pH (choice C), provide further evidence as to the cause of the patient’s
particularly if it grossly appears to be pus, is character- subclinical hypothyroidism. They are not necessary in
istic of an empyema, not heart failure. Rheumatoid determining whether to treat this patient.
arthritis also may cause a low glucose effusion. Isolated T3 deficiency is extremely rare and unlikely to
An elevated LDH level in the pleural fluid (choice D) explain this patient’s abnormal laboratory values.
suggests an exudate. If the level is greater than two Further, T3 supplementation is dangerous because of its
thirds the upper limit of normal for serum LDH or the variable bioavailability and ability to induce cardiac
ratio of pleural to serum LDH is >0.6, the fluid is arrhythmias, and it would not be a good agent for an
exudative. asymptomatic patient. There is no need to order a T3
A total protein level similar to serum protein (choice E) level or start supplemental T3 treatment (choice D).
makes the effusion exudative. A ratio of pleural protein Close followup, perhaps with an appointment in 3
to serum protein of >0.5 characterizes the effusion as months (choice E), is appropriate if this patient does
an exudate. not wish to start T4 treatment. In this patient, however,
treatment is beneficial and should be started if the
patient so desires.

57
USMLE Step 3 Assessment Exam

68. The correct answer is A. This patient is in acute Withholding treatment at this time (choice E) is not the
pulmonary edema in the setting of rapid atrial fibrilla- best course of action. This patient should be treated
tion that is now causing hypotension as well. Her now to avoid further spread of the infection. He also
increased heart rate is causing diminished ventricular should inform his sexual partners of his infection so
filling. It is not known how long she has been in atrial that they can be treated. He needs counseling about safe
fibrillation. The immediate goal of treatment must be sex practices for the future.
to slow the heart rate or to restore sinus rhythm. Given
the life-threatening presentation, urgent cardioversion 70. The correct answer is C. This patient has a classic pre-
is indicated. She must also be anticoagulated to prevent sentation of a subarachnoid hemorrhage, which often
thromboembolic episodes after the event. occurs from a ruptured saccular aneurysm (berry
aneurysm). The sentinal headaches, family history, and
Intravenous metoprolol (choice B) would have been
sudden development of symptoms are classic. A brief
useful had the patient not been decompensating.
period of loss of consciousness or the occurrence of
However, a beta-blocker is relatively contraindicated in
seizures is not uncommon. CT scan may miss an acute
a patient with chronic obstructive pulmonary disease.
bleed in up to 10% of patients. If there is a high index
Intravenous digoxin (choice C) may reduce the ventric- of suspicion, a lumbar puncture should be performed,
ular rate slightly. However, given its vagal mode of func- which will reveal xanthochromic CSF (pink or yellow
tion, the rate is unlikely to be controlled as effectively. tinged from breakdown of red blood cells).
Intravenous diltiazem (choice D) would be useful in rapid Cerebral angiography (choice A) is almost never per-
atrial fibrillation if the patient were not in pulmonary formed, particularly at 4 AM. If a CT scan and a lumbar
edema or hypotensive. It is in fact an effective modality to puncture are negative, it is a test to consider.
control rapid atrial fibrillation in a stable patient.
A CT scan with contrast (choice B) should not be per-
Intravenous adenosine (choice E) will be effective only formed if there is concern for an acute bleed. A non-
transiently. It would be helpful to clarify the rhythm contrast study is sensitive for bleeding and avoids the
and distinguish it from multifocal atrial tachycardia. risk for extravasation.
69. The correct answer is B. This patient likely has urethri- Magnetic resonance imaging (choice D) is less sensitive
tis. Urethritis is classified as gonococcal or nongonococ- than a CT scan for early bleeds. If this patient presented
cal. Gonococcal urethritis is caused by Neisseria gonor- later, an MRI might be appropriate. At this point, how-
rhoeae. Nongonococcal disease is caused by Chlamydia ever, a lumbar puncture is more likely to confirm the
trachomatis, Ureaplasma urealyticum, Mycoplasma homi- diagnosis.
nis, and Trichomonas. The best treatment for this patient A radiograph (choice E) does not provide more infor-
is to treat empirically with a single dose of ceftriaxone mation than the CT scan and is generally only useful if
and azithromycin. This regimen covers gonococcal and a skull fracture or bony lesion is suspected, in which
nongonococcal infections. Patients are often infected case it may be superior to a CT scan.
with both organisms, so single-dose treat for both to
insure compliance. 71. The correct answer is C. As a result of his major trauma,
this patient is suffering from disseminated intravascular
Treatment for gonorrhea alone (choice A) may miss a
coagulation (DIC). Large amounts of trauma to vascu-
significant number of cases of coexistent nongonococ-
lar beds and release of cytokines related to the inflam-
cal urethritis. Treatment for both infections helps lessen
matory reaction can lead to massive overactivation of
the chance of further disease spread and it is therefore
the coagulation cascade. Excessive conversion of
best to treat patients empirically for both infections.
prothrombin to active thrombin also leads to activation
Prophylaxis for HIV (choice C) is not indicated. of antithrombin III to counterbalance the coagulation
Patients with needle sticks can receive prophylaxis for cascade. Antithrombin III, though, is quickly consumed,
HIV within 25 hours of the incident, but there is no leaving thrombin unopposed. Depletion of antithrom-
evidence that prophylaxis several days after an unpro- bin III seems to be the crucial event in the initiation of
tected sexual intercourse effectively decreases transmis- DIC. This depletion leads to formation of diffuse
sion of HIV. This patient should receive an HIV test microthrombi, resulting in vessel occlusion and end-
now and in 6 months. organ ischemia. Such multiple organ failure is evident in
This patient does not need treatment for syphilis this patient who has developed acute renal failure and
(choice D). He has no lesions consistent with syphilis at respiratory compromise. After this initial hypercoagula-
this time. He should, however, receive a screening test tive state, clotting factors and platelets are consumed,
for syphilis. leading to uncontrolled bleeding as noted in this patient.

58
Internal Medicine Answers and Explanations

Surprisingly, fibrinogen levels may be normal or only patients with “red flags” such as anemia, weight loss,
slightly decreased in DIC. This stems from the fact that abdominal pain, or evidence of gastrointestinal blood
patients are invariably fighting another source of loss.
inflammation, such as infection, trauma, surgery, or tox-
emia. Because fibrinogen is an acute phase reactant, its 73. The correct answer is A. Chronic transfusion therapy
levels initially are elevated as the patient mounts an results in a variety of long-term complications. One of
inflammatory response. As DIC progresses, fibrinogen is these complications is the development of alloanti-
consumed together with the other clotting factors, but is bodies—antibodies that are directed against minor
not depleted to the same extent because of the initially antigen groups on transfused red blood cells. While the
high levels. presence of these antibodies may or may not produce
frank hemolysis, they will shorten the life of transfused
Activated partial thromboplastin time and prothrom-
blood and complicate locating appropriate blood. The
bin time (choices A and E) are incorrect because by this
blood bank performs a cross-match that needs to be
time, the patient has consumed large quantities of
negative (no coagulation or hemolysis) except in the
coagulation factors. A prolonged PT and aPTT are
case of emergencies.
almost universally elevated in DIC, except in the earli-
est stages of the disease. This patient has started to bleed Autoantibodies (choice B) do not explain the difficulty
and so is clearly in the later stages. in finding blood for this patient. Chronic transfusions
result in immune sensitization against transfused (for-
Antithrombin levels (choice B) is incorrect because, as
eign) blood, not in the development of autoantibodies.
stated, the depletion of antithrombin III is likely the
event that begins the downward spiral of DIC. Patients who are planned to undergo transplantation or
who have had transfusion reactions will often have spe-
Platelet count (choice D) is incorrect because although
cially filtered blood. This patient has no need for special
platelets are also acute phase reactants, their levels do
filtering of the blood (choice C) that would explain the
not increase as dramatically early in acute inflamma-
need to look far and wide for an appropriate specimen.
tion. Also, they are consumed more rapidly than is fib-
rinogen, and so patients in DIC always develop some Anti-HLA antibodies (choice D) are antibodies against
degree of thrombocytopenia. major blood groups. It is extremely unlikely that a
blood bank would not be able to quickly match a
72. The correct answer is C. Microcytic anemia and con- patient’s HLA type.
stipation in a man who has presumably not had age- Progression of this patient’s myelodysplasia (choice E)
appropriate colon cancer screening must be evaluated does not explain the difficulty in finding appropriate
for colon cancer. Given this patient’s anemia, a full blood for him. Progression of the disease results in less
colonoscopy, which can visualize a significantly greater production of red blood cells but does not explain why
area of the colon than a flexible sigmoidoscopy (choice it is difficult to find appropriate blood for a transfusion.
B), is an appropriate next step.
Fecal occult blood testing (choice A) has utility as a 74. The correct answer is B. This patient has polyarteritis
screening test in asymptomatic patients. However, given nodosa (PAN), a vasculitis strongly associated with hep-
that this patient has symptoms that are consistent with atitis B. Multiorgan system involvement should suggest
colon cancer, such as microcytic anemia and constipa- a vasculitis. Among the different types of vasculitis, this
tion, a full colonoscopy is warranted. Regardless of the patient’s symptoms are consistent with PAN. Most
results of the fecal occult blood testing, which can have patients complain of systemic symptoms such as weight
variable sensitivity and specificity, this patient will need loss, malaise, and fever. Additionally, approximately half
a colonoscopy, rendering the stool test results irrelevant. have evidence of neuropathy, commonly mononeuritis
multiplex or polyneuropathy. Diffuse arthralgias and
Senna (choice D) is a stimulant laxative that should be
myalgias are common. Commonly, renal involvement is
used for the short-term treatment of constipation. It is
also present, evidenced by an increased creatinine,
not an appropriate long-term therapy for constipation
hematuria, or glomerulonephritis. Skin involvement
because patients can become stimulant-dependent, and
often takes the form of livedo reticularis, seen in this
it is certainly not an appropriate treatment for someone
patient. Other clues include new-onset hypertension,
who may have more significant pathology.
stroke or coronary disease in an otherwise low-risk
A trial of fiber, fluids, and stool softeners (choice E) is patient, orchitis, and unexplained pulmonary infiltrates.
an appropriate treatment for constipation once organic
causes of diseases are excluded. It is not appropriate in

59
USMLE Step 3 Assessment Exam

Hepatitis A (choice A) is an acute, self-limiting disease


and is not associated with long-term systemic condi-
tions. It is not associated with any common forms of
vasculitis.
Hepatitis C (choice C) is associated with the formation
of cryoglobulins. Cryoglobulinemia can cause a similar
condition. The main distinguishing feature of cryo-
globulinemia is palpable purpura seen commonly on
the lower extremities, though it may also cause renal
disease, arthralgias, and other systemic manifestations.
Hepatitis D (choice D) can cause fulminant hepatic
failure when it occurs as a coinfection with hepatitis B.
It does not cause these symptoms of vasculitis.
Hepatitis E (choice E) is rare in the United States and
generally causes an acute, self-limited hepatitis, though it
may cause fulminant hepatic failure in pregnant women.

75. The correct answer is C. This patient has a right ventri-


cle infarction, as evidenced by the fact that her left atrial
pressure is lower than that in the right atrium. She is
exquisitely preload-dependent. At this time, the course of
action should include aggressive fluid resuscitation so
that her wedge pressure starts coming up.
Positive inotropic agents in this patient may be required
in the form of digoxin (choice A) as this patient’s med-
ical course progresses. This is not required in the acute
setting, however.
Similarly, dobutamine (choice B) is not required
before fluid resuscitation. If the patient’s cardiac out-
put is diminished, however, this agent may be
required. Care must be taken to ensure that her
ischemia is not worsened.
Because this patient is exquisitely preload-dependent,
using a nitric oxide derivative such as isosorbide
mononitrate (choice D) is not advisable because
preload decreases as a result of vasodilation.
If the patient does not respond to the initial medical
intervention, afterload reduction may be required in
the form of the placement of an intraaortic balloon
pump (choice E).

60
BM3003A Printed in USA © 2004

kaplanmedical.com

Anda mungkin juga menyukai